Solve a Hidato puzzle: Difference between revisions

From Rosetta Code
Content added Content deleted
(→‎{{header|Perl 6}}: Syntax fix, "0" is true now. (didn't actually affect this particular problem))
(Added Go)
Line 1,352: Line 1,352:
17 7 6 3
17 7 6 3
5 4
5 4
</pre>

=={{header|Go}}==
{{trans|Java}}
<lang go>package main

import (
"fmt"
"sort"
"strconv"
"strings"
)

var board [][]int
var start, given []int

func setup(input []string) {
/* This task is not about input validation, so
we're going to trust the input to be valid */
puzzle := make([][]string, len(input))
for i := 0; i < len(input); i++ {
puzzle[i] = strings.Fields(input[i])
}
nCols := len(puzzle[0])
nRows := len(puzzle)
list := make([]int, nRows*nCols)
board = make([][]int, nRows+2)
for i := 0; i < nRows+2; i++ {
board[i] = make([]int, nCols+2)
for j := 0; j < nCols+2; j++ {
board[i][j] = -1
}
}
for r := 0; r < nRows; r++ {
row := puzzle[r]
for c := 0; c < nCols; c++ {
switch cell := row[c]; cell {
case "_":
board[r+1][c+1] = 0
case ".":
break
default:
val, _ := strconv.Atoi(cell)
board[r+1][c+1] = val
list = append(list, val)
if val == 1 {
start = append(start, r+1, c+1)
}
}
}
}
sort.Ints(list)
given = make([]int, len(list))
for i := 0; i < len(given); i++ {
given[i] = list[i]
}
}

func solve(r, c, n, next int) bool {
if n > given[len(given)-1] {
return true
}

back := board[r][c]
if back != 0 && back != n {
return false
}

if back == 0 && given[next] == n {
return false
}

if back == n {
next++
}

board[r][c] = n
for i := -1; i < 2; i++ {
for j := -1; j < 2; j++ {
if solve(r+i, c+j, n+1, next) {
return true
}
}
}

board[r][c] = back
return false
}

func printBoard() {
for _, row := range board {
for _, c := range row {
switch {
case c == -1:
fmt.Print(" . ")
case c > 0:
fmt.Printf("%2d ", c)
default:
fmt.Print("__ ")
}
}
fmt.Println()
}
}

func main() {
input := []string{
"_ 33 35 _ _ . . .",
"_ _ 24 22 _ . . .",
"_ _ _ 21 _ _ . .",
"_ 26 _ 13 40 11 . .",
"27 _ _ _ 9 _ 1 .",
". . _ _ 18 _ _ .",
". . . . _ 7 _ _",
". . . . . . 5 _",
}
setup(input)
printBoard()
fmt.Println("\nFound:")
solve(start[0], start[1], 1, 0)
printBoard()
}</lang>

{{out}}
<pre>
. . . . . . . . . .
. __ 33 35 __ __ . . . .
. __ __ 24 22 __ . . . .
. __ __ __ 21 __ __ . . .
. __ 26 __ 13 40 11 . . .
. 27 __ __ __ 9 __ 1 . .
. . . __ __ 18 __ __ . .
. . . . . __ 7 __ __ .
. . . . . . . 5 __ .
. . . . . . . . . .

Found:
. . . . . . . . . .
. 32 33 35 36 37 . . . .
. 31 34 24 22 38 . . . .
. 30 25 23 21 12 39 . . .
. 29 26 20 13 40 11 . . .
. 27 28 14 19 9 10 1 . .
. . . 15 16 18 8 2 . .
. . . . . 17 7 6 3 .
. . . . . . . 5 4 .
. . . . . . . . . .
</pre>
</pre>



Revision as of 14:33, 27 June 2018

Task
Solve a Hidato puzzle
You are encouraged to solve this task according to the task description, using any language you may know.

The task is to write a program which solves Hidato (aka Hidoku) puzzles.

The rules are:

  • You are given a grid with some numbers placed in it. The other squares in the grid will be blank.
    • The grid is not necessarily rectangular.
    • The grid may have holes in it.
    • The grid is always connected.
    • The number “1” is always present, as is another number that is equal to the number of squares in the grid. Other numbers are present so as to force the solution to be unique.
    • It may be assumed that the difference between numbers present on the grid is not greater than lucky 13.
  • The aim is to place a natural number in each blank square so that in the sequence of numbered squares from “1” upwards, each square is in the wp:Moore neighborhood of the squares immediately before and after it in the sequence (except for the first and last squares, of course, which only have one-sided constraints).
    • Thus, if the grid was overlaid on a chessboard, a king would be able to make legal moves along the path from first to last square in numerical order.
    • A square may only contain one number.
  • In a proper Hidato puzzle, the solution is unique.


For example the following problem

Sample Hidato problem, from Wikipedia
Sample Hidato problem, from Wikipedia

has the following solution, with path marked on it:

Solution to sample Hidato problem
Solution to sample Hidato problem


Related tasks



AutoHotkey

<lang AutoHotkey>SolveHidato(Grid, Locked, Max, row, col, num:=1, R:="", C:=""){ if (R&&C) ; if neighbors (not first iteration) { Grid[R, C] := ">" num ; place num in current neighbor and mark it visited ">" row:=R, col:=C ; move to current neighbor }

num++ ; increment num if (num=max) ; if reached end return map(Grid) ; return solution

if locked[num] ; if current num is a locked value { row := StrSplit((StrSplit(locked[num], ",").1) , ":").1 ; find row of num col := StrSplit((StrSplit(locked[num], ",").1) , ":").2 ; find col of num if SolveHidato(Grid, Locked, Max, row, col, num) ; solve for current location and value return map(Grid) ; if solved, return solution } else { for each, value in StrSplit(Neighbor(row,col), ",") { R := StrSplit(value, ":").1 C := StrSplit(value, ":").2

if (Grid[R,C] = "") ; a hole or out of bounds || InStr(Grid[R, C], ">") ; visited || Locked[num+1] && !(Locked[num+1]~= "\b" R ":" C "\b") ; not neighbor of locked[num+1] || Locked[num-1] && !(Locked[num-1]~= "\b" R ":" C "\b") ; not neighbor of locked[num-1] || Locked[num] ; locked value || Locked[Grid[R, C]] ; locked cell continue

if SolveHidato(Grid, Locked, Max, row, col, num, R, C) ; solve for current location, neighbor and value return map(Grid) ; if solved, return solution } } num-- ; step back for i, line in Grid for j, element in line if InStr(element, ">") && (StrReplace(element, ">") >= num) Grid[i, j] := "Y" }

--------------------------------
--------------------------------
--------------------------------

Neighbor(row,col){ R := row-1 loop, 9 { DeltaC := Mod(A_Index, 3) ? Mod(A_Index, 3)-2 : 1 res .= (R=row && !DeltaC) ? "" : R ":" col+DeltaC "," R := Mod(A_Index, 3) ? R : R+1 } return Trim(res, ",") }

--------------------------------

map(Grid){ for i, row in Grid { for j, element in row line .= (A_Index > 1 ? "`t" : "") . element map .= (map<>""?"`n":"") line line := "" } return StrReplace(map, ">") }</lang> Examples:<lang AutoHotkey>;-------------------------------- Grid := [[ "Y" , 33 , 35 , "Y" , "Y"] ,[ "Y" , "Y" , 24 , 22 , "Y"] ,[ "Y" , "Y" , "Y" , 21 , "Y" , "Y"] ,[ "Y" , 26 , "Y" , 13 , 40 , 11 ] ,[ 27 , "Y" , "Y" , "Y" , 9 , "Y" , 1 ] ,[ "" , "" , "Y" , "Y" , 18 , "Y" , "Y"] ,[ "" , "" , "" , "" , "Y" , 7 , "Y" , "Y"] ,[ "" , "" , "" , "" , "" , "" , 5 , "Y"]]

--------------------------------
find locked cells, find row and col of first value "1" and max value

Locked := [] for i, line in Grid for j, element in line { if element = 1 row :=i , col := j if element is integer Locked[element] := i ":" j "," Neighbor(i, j) ; save locked elements position and neighbors , max := element > max ? element : max ; find max value }

--------------------------------

MsgBox, 262144, ,% SolveHidato(Grid, Locked, Max, row, col) return</lang>

Outputs:

32	33	35	36	37
31	34	24	22	38
30	25	23	21	12	39
29	26	20	13	40	11
27	28	14	19	9	10	1
		15	16	18	8	2
				17	7	6	3
						5	4

Bracmat

<lang bracmat>(

 ( hidato
 =     Line solve lowest Ncells row column rpad
     , Board colWidth maxDigits start curCol curRow
     , range head line cellN solution output tail
   .   out$!arg
     & @(!arg:? ((%@:>" ") ?:?arg))
     & 0:?row:?column
     & :?Board
     & ( Line
       =   token
         .   whl
           ' ( @(!arg:?token [3 ?arg)
             & (   (   @(!token:? "_" ?)
                     & :?token
                   | @(!token:? #?token (|" " ?))
                   )
                 & (!token.!row.!column) !Board:?Board
               | 
               )
             & 1+!column:?column
             )
       )
     &   whl
       ' ( @(!arg:?line \n ?arg)
         & Line$!line
         & 1+!row:?row
         & 0:?column
         )
     & Line$!arg
     & ( range
       =   hi lo
         .   (!arg+1:?hi)+-2:?lo
           & '($lo|$arg|$hi)
       )
     & ( solve
       =     ToDo cellN row column head tail remainder
           , candCell Solved rowCand colCand pattern recurse
         .   !arg:(?ToDo.?cellN.?row.?column)
           & range$!row:(=?row)
           & range$!column:(=?column)
           &     
               ' (     ?head ($cellN.?rowCand.?colCand) ?tail
                     & (!rowCand.!colCand):($row.$column)
                     & !recurse
                   |   ?head
                       (.($row.$column):(?rowCand.?colCand))
                       (?tail&!recurse)
                 .     ((!rowCand.!colCand).$cellN)
                     : ?candCell
                   &   (   !head !tail:
                         & out$found!
                         & !candCell
                       |       solve
                             $ ( !head !tail
                               . $cellN+1
                               . !rowCand
                               . !colCand
                               )
                           : ?remainder
                         & !candCell+!remainder
                       )
                     : ?Solved
                 )
             : (=?pattern.?recurse)
           & !ToDo:!pattern
           & !Solved
       )
     & infinity:?lowest
     & (   !Board
         : ? (<!lowest:#%?lowest.?start) (?&~)
       | solve$(!Board.!lowest.!start):?solution
       )
     & :?output
     & 0:?curCol
     & !solution:((?curRow.?).?)+?+[?Ncells
     & @(!Ncells:? [?maxDigits)
     & 1+!maxDigits:?colWidth
     & ( rpad
       =   len
         .   !arg:(?arg.?len)
           & @(str$(!arg "    "):?arg [!len ?)
           & !arg
       )
     &   whl
       ' ( !solution:((?row.?column).?cellN)+?solution
         & (   !row:>!curRow:?curRow
             & !output \n:?output
             & 0:?curCol
           | 
           )
         &   whl
           ' ( !curCol+1:~>!column:?curCol
             & !output rpad$(.!colWidth):?output
             )
         &   !output rev$(rpad$(rev$(str$(!cellN " ")).!colWidth))
           : ?output
         & !curCol+1:?curCol
         )
     & str$!output
 )

& "

__ 33 35 __ __         
__ __ 24 22 __         
__ __ __ 21 __ __      
__ 26 __ 13 40 11      
27 __ __ __  9 __  1   
      __ __ 18 __ __   
            __  7 __ __
                   5 __"
 : ?board

& out$(hidato$!board) );</lang> Output:


 __ 33 35 __ __
 __ __ 24 22 __
 __ __ __ 21 __ __
 __ 26 __ 13 40 11
 27 __ __ __  9 __  1
       __ __ 18 __ __
             __  7 __ __
                    5 __
found!
32 33 35 36 37
31 34 24 22 38
30 25 23 21 12 39
29 26 20 13 40 11
27 28 14 19  9 10  1
      15 16 18  8  2
            17  7  6  3
                   5  4

C

Depth-first graph, with simple connectivity check to reject some impossible situations early. The checks slow down simpler puzzles significantly, but can make some deep recursions backtrack much earilier. <lang c>#include <stdio.h>

  1. include <stdlib.h>
  2. include <string.h>
  3. include <ctype.h>

int *board, *flood, *known, top = 0, w, h;

static inline int idx(int y, int x) { return y * w + x; }

int neighbors(int c, int *p) /* @c cell @p list of neighbours @return amount of neighbours

  • /

{ int i, j, n = 0; int y = c / w, x = c % w;

for (i = y - 1; i <= y + 1; i++) { if (i < 0 || i >= h) continue; for (j = x - 1; j <= x + 1; j++) if (!(j < 0 || j >= w || (j == x && i == y) || board[ p[n] = idx(i,j) ] == -1)) n++; }

return n; }

void flood_fill(int c) /* fill all free cells around @c with “1” and write output to variable “flood” @c cell

  • /

{ int i, n[8], nei;

nei = neighbors(c, n); for (i = 0; i < nei; i++) { // for all neighbours if (board[n[i]] || flood[n[i]]) continue; // if cell is not free, choose another neighbour

flood[n[i]] = 1; flood_fill(n[i]); } }

/* Check all empty cells are reachable from higher known cells.

  Should really do more checks to make sure cell_x and cell_x+1
  share enough reachable empty cells; I'm lazy. Will implement
  if a good counter example is presented. */

int check_connectity(int lowerbound) { int c; memset(flood, 0, sizeof(flood[0]) * w * h); for (c = lowerbound + 1; c <= top; c++) if (known[c]) flood_fill(known[c]); // mark all free cells around known cells

for (c = 0; c < w * h; c++) if (!board[c] && !flood[c]) // if there are free cells which could not be reached from flood_fill return 0;

return 1; }

void make_board(int x, int y, const char *s) { int i;

w = x, h = y;

       top = 0;

x = w * h;

       known = calloc(x + 1, sizeof(int));
       board = calloc(x,     sizeof(int));
       flood = calloc(x,     sizeof(int));

while (x--) board[x] = -1;

for (y = 0; y < h; y++) for (x = 0; x < w; x++) { i = idx(y, x);

while (isspace(*s)) s++;

switch (*s) { case '_': board[i] = 0; case '.': break; default: known[ board[i] = strtol(s, 0, 10) ] = i; if (board[i] > top) top = board[i]; }

while (*s && !isspace(*s)) s++; } }

void show_board(const char *s) { int i, j, c;

printf("\n%s:\n", s);

for (i = 0; i < h; i++, putchar('\n')) for (j = 0; j < w; j++) { c = board[ idx(i, j) ]; printf(!c ? " __" : c == -1 ? " " : " %2d", c); } }

int fill(int c, int n) { int i, nei, p[8], ko, bo;

if ((board[c] && board[c] != n) || (known[n] && known[n] != c)) return 0;

if (n == top) return 1;

ko = known[n]; bo = board[c]; board[c] = n;

if (check_connectity(n)) { nei = neighbors(c, p); for (i = 0; i < nei; i++) if (fill(p[i], n + 1)) return 1; }

board[c] = bo; known[n] = ko; return 0; }

int main() { make_board(

  1. define USE_E 0
  2. if (USE_E == 0)

8,8, " __ 33 35 __ __ .. .. .." " __ __ 24 22 __ .. .. .." " __ __ __ 21 __ __ .. .." " __ 26 __ 13 40 11 .. .." " 27 __ __ __ 9 __ 1 .." " . . __ __ 18 __ __ .." " . .. . . __ 7 __ __" " . .. .. .. . . 5 __"

  1. elif (USE_E == 1)

3, 3, " . 4 ." " _ 7 _" " 1 _ _"

  1. else

50, 3, " 1 _ _ . . _ _ . . _ _ . . _ _ . . _ _ . . _ _ . . _ _ . . _ _ . . _ _ . . _ _ . . _ _ . . _ _ . . 74" " . . _ . _ . _ . _ . _ . _ . _ . _ . _ . _ . _ . _ . _ . _ . _ . _ . _ . _ . _ . _ . _ . _ . _ . _ ." " . . . _ _ . . _ _ . . _ _ . . _ _ . . _ _ . . _ _ . . _ _ . . _ _ . . _ _ . . _ _ . . _ _ . . _ _ ."

  1. endif

);

show_board("Before"); fill(known[1], 1); show_board("After"); /* "40 lbs in two weeks!" */

return 0; }</lang>

Output:
 Before:
 __ 33 35 __ __         
 __ __ 24 22 __         
 __ __ __ 21 __ __      
 __ 26 __ 13 40 11      
 27 __ __ __  9 __  1   
       __ __ 18 __ __   
             __  7 __ __
                    5 __
 
 After:
 32 33 35 36 37         
 31 34 24 22 38         
 30 25 23 21 12 39      
 29 26 20 13 40 11      
 27 28 14 19  9 10  1   
       15 16 18  8  2   
             17  7  6  3
                    5  4

C++

<lang cpp>

  1. include <iostream>
  2. include <sstream>
  3. include <iterator>
  4. include <vector>

//------------------------------------------------------------------------------ using namespace std;

//------------------------------------------------------------------------------ struct node {

   int val;
   unsigned char neighbors;

}; //------------------------------------------------------------------------------ class hSolver { public:

   hSolver()
   {

dx[0] = -1; dx[1] = 0; dx[2] = 1; dx[3] = -1; dx[4] = 1; dx[5] = -1; dx[6] = 0; dx[7] = 1; dy[0] = -1; dy[1] = -1; dy[2] = -1; dy[3] = 0; dy[4] = 0; dy[5] = 1; dy[6] = 1; dy[7] = 1;

   }
   void solve( vector<string>& puzz, int max_wid )
   {

if( puzz.size() < 1 ) return; wid = max_wid; hei = static_cast<int>( puzz.size() ) / wid; int len = wid * hei, c = 0; max = 0; arr = new node[len]; memset( arr, 0, len * sizeof( node ) ); weHave = new bool[len + 1]; memset( weHave, 0, len + 1 );

for( vector<string>::iterator i = puzz.begin(); i != puzz.end(); i++ ) { if( ( *i ) == "*" ) { arr[c++].val = -1; continue; } arr[c].val = atoi( ( *i ).c_str() ); if( arr[c].val > 0 ) weHave[arr[c].val] = true; if( max < arr[c].val ) max = arr[c].val; c++; }

solveIt(); c = 0; for( vector<string>::iterator i = puzz.begin(); i != puzz.end(); i++ ) { if( ( *i ) == "." ) { ostringstream o; o << arr[c].val; ( *i ) = o.str(); } c++; } delete [] arr; delete [] weHave;

   }

private:

   bool search( int x, int y, int w )
   {

if( w == max ) return true;

node* n = &arr[x + y * wid]; n->neighbors = getNeighbors( x, y ); if( weHave[w] ) { for( int d = 0; d < 8; d++ ) { if( n->neighbors & ( 1 << d ) ) { int a = x + dx[d], b = y + dy[d]; if( arr[a + b * wid].val == w ) if( search( a, b, w + 1 ) ) return true; } } return false; }

for( int d = 0; d < 8; d++ ) { if( n->neighbors & ( 1 << d ) ) { int a = x + dx[d], b = y + dy[d]; if( arr[a + b * wid].val == 0 ) { arr[a + b * wid].val = w; if( search( a, b, w + 1 ) ) return true; arr[a + b * wid].val = 0; } } } return false;

   }
   unsigned char getNeighbors( int x, int y )
   {

unsigned char c = 0; int m = -1, a, b; for( int yy = -1; yy < 2; yy++ ) for( int xx = -1; xx < 2; xx++ ) { if( !yy && !xx ) continue; m++; a = x + xx, b = y + yy; if( a < 0 || b < 0 || a >= wid || b >= hei ) continue; if( arr[a + b * wid].val > -1 ) c |= ( 1 << m ); } return c;

   }
   void solveIt()
   {

int x, y; findStart( x, y ); if( x < 0 ) { cout << "\nCan't find start point!\n"; return; } search( x, y, 2 );

   }
   void findStart( int& x, int& y )
   {

for( int b = 0; b < hei; b++ ) for( int a = 0; a < wid; a++ ) if( arr[a + wid * b].val == 1 ) { x = a; y = b; return; } x = y = -1;

   }
   int wid, hei, max, dx[8], dy[8];
   node* arr;
   bool* weHave;

}; //------------------------------------------------------------------------------ int main( int argc, char* argv[] ) {

   int wid;
   string p = ". 33 35 . . * * * . . 24 22 . * * * . . . 21 . . * * . 26 . 13 40 11 * * 27 . . . 9 . 1 * * * . . 18 . . * * * * * . 7 . . * * * * * * 5 ."; wid = 8;
   //string p = "54 . 60 59 . 67 . 69 . . 55 . . 63 65 . 72 71 51 50 56 62 . * * * * . . . 14 * * 17 . * 48 10 11 * 15 . 18 . 22 . 46 . * 3 . 19 23 . . 44 . 5 . 1 33 32 . . 43 7 . 36 . 27 . 31 42 . . 38 . 35 28 . 30"; wid = 9;
   //string p = ". 58 . 60 . . 63 66 . 57 55 59 53 49 . 65 . 68 . 8 . . 50 . 46 45 . 10 6 . * * * . 43 70 . 11 12 * * * 72 71 . . 14 . * * * 30 39 . 15 3 17 . 28 29 . . 40 . . 19 22 . . 37 36 . 1 20 . 24 . 26 . 34 33"; wid = 9;
   istringstream iss( p ); vector<string> puzz;
   copy( istream_iterator<string>( iss ), istream_iterator<string>(), back_inserter<vector<string> >( puzz ) );
   hSolver s; s.solve( puzz, wid );
   int c = 0;
   for( vector<string>::iterator i = puzz.begin(); i != puzz.end(); i++ )
   {

if( ( *i ) != "*" && ( *i ) != "." ) { if( atoi( ( *i ).c_str() ) < 10 ) cout << "0"; cout << ( *i ) << " "; } else cout << " "; if( ++c >= wid ) { cout << endl; c = 0; }

   }
   cout << endl << endl;
   return system( "pause" );

} //-------------------------------------------------------------------------------------------------- </lang> Output:

32 33 35 36 37
31 34 24 22 38
30 25 23 21 12 39
29 26 20 13 40 11
27 28 14 19 09 10 01
      15 16 18 08 02
            17 07 06 03
                  05 04

56 58 54 60 61 62 63 66 67
57 55 59 53 49 47 65 64 68
09 08 52 51 50 48 46 45 69
10 06 07          44 43 70
05 11 12          72 71 42
04 14 13          30 39 41
15 03 17 18 28 29 38 31 40
02 16 19 22 23 27 37 36 32
01 20 21 24 25 26 35 34 33

Curry

Works with: PAKCS

Probably not efficient. <lang curry>import CLPFD import Constraint (andC, anyC) import Findall (unpack) import Integer (abs)


hidato :: Int -> Success hidato path =

   test path inner
 & domain inner 1 40
 & allDifferent inner
 & andFD [x `near` y | x <- cells, y <- cells]
 & labeling [] (concat path)
 where
   andFD = solve . foldr1 (#/\#)
   cells = enumerate path
   inner free

near :: (Int,Int,Int) -> (Int,Int,Int) -> Constraint (x,rx,cx) `near` (y,ry,cy) = x #<=# y #/\# dist (y -# x)

                       #\/#  x #>#  y  #/\#  dist (x -# y)
                       #\/#  x #=#  0
                       #\/#  y #=#  0
 where
   dist d =  abs (rx - ry) #<=# d
       #/\#  abs (cx - cy) #<=# d

enumerate :: Int -> [(Int,Int,Int)] enumerate xss = [(x,row,col) | (xs,row) <- xss `zip` [1..]

                            , (x ,col) <- xs  `zip` [1..]
               ]

test [[ 0, 0, 0, 0, 0, 0, 0, 0, 0, 0]

    ,[ 0,  A, 33, 35,  B,  C,  0, 0, 0, 0]
    ,[ 0,  D,  E, 24, 22,  F,  0, 0, 0, 0]
    ,[ 0,  G,  H,  I, 21,  J,  K, 0, 0, 0]
    ,[ 0,  L, 26,  M, 13, 40, 11, 0, 0, 0]
    ,[ 0, 27,  N,  O,  P,  9,  Q, 1, 0, 0]
    ,[ 0,  0,  0,  R,  S, 18,  T, U, 0, 0]
    ,[ 0,  0,  0,  0,  0,  V,  7, W, X, 0]
    ,[ 0,  0,  0,  0,  0,  0,  0, 5, Y, 0]
    ,[ 0,  0,  0,  0,  0,  0,  0, 0, 0, 0]
    ]
    [ A, 33, 35,  B,  C
    , D,  E, 24, 22,  F
       , G,  H,  I, 21,  J,  K
       , L, 26,  M, 13, 40, 11
          , 27,  N,  O,  P,  9, Q, 1
          ,  R,  S, 18,  T,  U
              ,  V,  7,  W,  X
                      ,  5,  Y
    ] = success

main = unpack hidato</lang>

Output:
Execution time: 1440 msec. / elapsed: 2270 msec.
[[0,0,0,0,0,0,0,0,0,0],[0,32,33,35,36,37,0,0,0,0],[0,31,34,24,22,38,0,0,0,0],[0,30,25,23,21,12,39,0,0,0],[0,29,26,20,13,40,11,0,0,0],[0,27,28,14,19,9,10,1,0,0],[0,0,0,15,16,18,8,2,0,0],[0,0,0,0,0,17,7,6,3,0],[0,0,0,0,0,0,0,5,4,0],[0,0,0,0,0,0,0,0,0,0]]
More values? [y(es)/N(o)/a(ll)]

D

More C-Style Version

This version retains some of the characteristics of the original C version. It uses global variables, it doesn't enforce immutability and purity. This style is faster to write for prototypes, short programs or less important code, but in larger programs you usually want more strictness to avoid some bugs and increase long-term maintainability.

Translation of: C

<lang d>import std.stdio, std.array, std.conv, std.algorithm, std.string;

int[][] board; int[] given, start;

void setup(string s) {

   auto lines = s.splitLines;
   auto cols = lines[0].split.length;
   auto rows = lines.length;
   given.length = 0;
   board = new int[][](rows + 2, cols + 2);
   foreach (row; board)
       row[] = -1;
   foreach (r, row; lines) {
       foreach (c, cell; row.split) {
           switch (cell) {
               case "__":
                   board[r + 1][c + 1] = 0;
                   break;
               case ".":
                   break;
               default:
                   int val = cell.to!int;
                   board[r + 1][c + 1] = val;
                   given ~= val;
                   if (val == 1)
                       start = [r + 1, c + 1];
           }
       }
   }
   given.sort();

}

bool solve(int r, int c, int n, int next = 0) {

   if (n > given.back)
       return true;
   if (board[r][c] && board[r][c] != n)
       return false;
   if (board[r][c] == 0 && given[next] == n)
       return false;
   int back = board[r][c];
   board[r][c] = n;
   foreach (i; -1 .. 2)
       foreach (j; -1 .. 2)
           if (solve(r + i, c + j, n + 1, next + (back == n)))
               return true;
   board[r][c] = back;
   return false;

}

void printBoard() {

   foreach (row; board) {
       foreach (c; row)
           writef(c == -1 ? " . " : c ? "%2d " : "__ ", c);
       writeln;
   }

}

void main() {

   auto hi = "__ 33 35 __ __  .  .  .
               __ __ 24 22 __  .  .  .
               __ __ __ 21 __ __  .  .
               __ 26 __ 13 40 11  .  .
               27 __ __ __  9 __  1  .
                .  . __ __ 18 __ __  .
                .  .  .  . __  7 __ __
                .  .  .  .  .  .  5 __";
   hi.setup;
   printBoard;
   "\nFound:".writeln;
   solve(start[0], start[1], 1);
   printBoard;

}</lang>

Output:
 .  .  .  .  .  .  .  .  .  . 
 . __ 33 35 __ __  .  .  .  . 
 . __ __ 24 22 __  .  .  .  . 
 . __ __ __ 21 __ __  .  .  . 
 . __ 26 __ 13 40 11  .  .  . 
 . 27 __ __ __  9 __  1  .  . 
 .  .  . __ __ 18 __ __  .  . 
 .  .  .  .  . __  7 __ __  . 
 .  .  .  .  .  .  .  5 __  . 
 .  .  .  .  .  .  .  .  .  . 

Found:
 .  .  .  .  .  .  .  .  .  . 
 . 32 33 35 36 37  .  .  .  . 
 . 31 34 24 22 38  .  .  .  . 
 . 30 25 23 21 12 39  .  .  . 
 . 29 26 20 13 40 11  .  .  . 
 . 27 28 14 19  9 10  1  .  . 
 .  .  . 15 16 18  8  2  .  . 
 .  .  .  .  . 17  7  6  3  . 
 .  .  .  .  .  .  .  5  4  . 
 .  .  .  .  .  .  .  .  .  . 

Stronger Version

Translation of: C

This version uses a little stronger typing, performs tests a run-time with contracts, it doesn't use global variables, it enforces immutability and purity where possible, and produces a correct text output for both larger ad small boards. This style is more fit for larger programs, or when you want the code to be less bug-prone or a little more efficient.

With this coding style the changes in the code become less bug-prone, but also more laborious. This version is also faster, its total runtime is about 0.02 seconds or less. <lang d>import std.stdio, std.conv, std.ascii, std.array, std.string,

      std.algorithm, std.exception, std.range, std.typetuple;

struct Hidato {

   // alias Cell = RangedValue!(int, -1, int.max);
   alias Cell = int;
   alias Pos = size_t;
   enum : Cell { emptyCell = -1, unknownCell = 0 }
   immutable Cell boardMax;
   immutable size_t nCols, nRows;
   Cell[] board;
   Pos[] known;
   bool[] flood;
   this(in string input) pure @safe
   in {
       assert(!input.strip.empty);
   } out {
       assert(nCols > 0 && nRows > 0);
       immutable size = nCols * nRows;
       assert(board.length == size);
       assert(known.length == size + 1);
       assert(flood.length == size);
       assert(boardMax > 0 && boardMax <= size);
       assert(board.reduce!max == boardMax);
       assert(board.canFind(1) && board.canFind(boardMax));
       assert(flood.all!(f => f == 0));
       assert(known.all!(rc => rc >= 0 && rc < size));
       foreach (immutable i, immutable cell; board) {
           assert(cell == Hidato.emptyCell ||
                  cell == Hidato.unknownCell ||
                  (cell >= 1 && cell <= size));
           if (cell > 0)
               assert(i == known[size_t(cell)]);
       }
   } body {
       bool[Cell] pathSeen; // A set.
       immutable lines = input.splitLines;
       this.nRows = lines.length;
       this.nCols = lines[0].split.length;
       immutable size = nCols * nRows;
       this.board.length = size;
       this.board[] = emptyCell;
       this.known.length = size + 1;
       this.flood.length = size;
       auto boardMaxMutable = Cell.min;
       Pos i = 0;
       foreach (immutable row; lines) {
           assert(row.split.length == nCols,
                  text("Wrong cols n.: ", row.split.length));
           foreach (immutable cell; row.split) {
               switch (cell) {
                   case "_":
                       this.board[i] = Hidato.unknownCell;
                       break;
                   case ".":
                       this.board[i] = Hidato.emptyCell;
                       break;
                   default: // Known.
                       immutable val = cell.to!Cell;
                       enforce(val > 0, "Path numbers must be > 0.");
                       enforce(val !in pathSeen,
                               text("Duplicated path number: ", val));
                       pathSeen[val] = true;
                       this.board[i] = val;
                       this.known[val] = i;
                       boardMaxMutable = max(boardMaxMutable, val);
               }
               i++;
           }
       }
       this.boardMax = boardMaxMutable;
   }


   private Pos idx(in size_t r, in size_t c) const pure nothrow @safe @nogc {
       return r * nCols + c;
   }
   private uint nNeighbors(in Pos pos, ref Pos[8] neighbours)
   const pure nothrow @safe @nogc {
       immutable r = pos / nCols;
       immutable c = pos % nCols;
       typeof(return) n = 0;
       foreach (immutable sr; TypeTuple!(-1, 0, 1)) {
           immutable size_t i = r + sr; // Can wrap-around.
           if (i >= nRows)
               continue;
           foreach (immutable sc; TypeTuple!(-1, 0, 1)) {
               immutable size_t j = c + sc; // Can wrap-around.
               if ((sc != 0 || sr != 0) && j < nCols) {
                   immutable pos2 = idx(i, j);
                   neighbours[n] = pos2;
                   if (board[pos2] != Hidato.emptyCell)
                       n++;
               }
           }
       }
       return n;
   }
   /// Fill all free cells around 'cell' with true and write
   /// output to variable "flood".
   private void floodFill(in Pos pos) pure nothrow @safe @nogc {
       Pos[8] n = void;
       // For all neighbours.
       foreach (immutable i; 0 .. nNeighbors(pos, n)) {
           // If pos is not free, choose another neighbour.
           if (board[n[i]] || flood[n[i]])
               continue;
           flood[n[i]] = true;
           floodFill(n[i]);
       }
   }
   /// Check all empty cells are reachable from higher known cells.
   private bool checkConnectity(in uint lowerBound) pure nothrow @safe @nogc {
       flood[] = false;
       foreach (immutable i; lowerBound + 1 .. boardMax + 1)
           if (known[i])
               floodFill(known[i]);
       foreach (immutable i; 0 .. nCols * nRows)
           // If there are free cells which could not be
           // reached from floodFill.
           if (!board[i] && !flood[i])
               return false;
       return true;
   }
   private bool fill(in Pos pos, in uint n) pure nothrow @safe @nogc {
       if ((board[pos] && board[pos] != n) ||
           (known[n] && known[n] != pos))
           return false;
       if (n == boardMax)
           return true;
       immutable ko = known[n];
       immutable bo = board[pos];
       board[pos] = n;
       Pos[8] p = void;
       if (checkConnectity(n))
           foreach (immutable i; 0 .. nNeighbors(pos, p))
               if (fill(p[i], n + 1))
                   return true;
       board[pos] = bo;
       known[n] = ko;
       return false;
   }
   void solve() pure nothrow @safe @nogc
   in {
       assert(!known.empty);
   } body {
       fill(known[1], 1);
   }
   string toString() const pure {
       immutable d = [Hidato.emptyCell: ".",
                      Hidato.unknownCell: "_"];
       immutable form = "%" ~ text(boardMax.text.length + 1) ~ "s";
       string result;
       foreach (immutable r; 0 .. nRows) {
           foreach (immutable c; 0 .. nCols) {
               immutable cell = board[idx(r, c)];
               result ~= format(form, d.get(cell, cell.text));
           }
           result ~= "\n";
       }
       return result;
   }

}

void solveHidato(in string problem) {

   auto game = problem.Hidato;
   writeln("Problem:\n", game);
   game.solve;
   writeln("Solution:\n", game);

}

void main() {

   solveHidato(" _ 33 35  _  _  .  .  .
                 _  _ 24 22  _  .  .  .
                 _  _  _ 21  _  _  .  .
                 _ 26  _ 13 40 11  .  .
                27  _  _  _  9  _  1  .
                 .  .  _  _ 18  _  _  .
                 .  .  .  .  _  7  _  _
                 .  .  .  .  .  .  5  _");
   solveHidato(". 4 .
                _ 7 _
                1 _ _");
   solveHidato(

"1 _ _ . . _ _ . . _ _ . . _ _ . . _ _ . . _ _ . . _ _ . . _ _ . . _ _ . . _ _ . . _ _ . . _ _ . . 74

. . _ . _ . _ . _ . _ . _ . _ . _ . _ . _ . _ . _ . _ . _ . _ . _ . _ . _ . _ . _ . _ . _ . _ . _ .
. . . _ _ . . _ _ . . _ _ . . _ _ . . _ _ . . _ _ . . _ _ . . _ _ . . _ _ . . _ _ . . _ _ . . _ _ ."
   );

}</lang>

Output:
Problem:
  _ 33 35  _  _  .  .  .
  _  _ 24 22  _  .  .  .
  _  _  _ 21  _  _  .  .
  _ 26  _ 13 40 11  .  .
 27  _  _  _  9  _  1  .
  .  .  _  _ 18  _  _  .
  .  .  .  .  _  7  _  _
  .  .  .  .  .  .  5  _

Solution:
 32 33 35 36 37  .  .  .
 31 34 24 22 38  .  .  .
 30 25 23 21 12 39  .  .
 29 26 20 13 40 11  .  .
 27 28 14 19  9 10  1  .
  .  . 15 16 18  8  2  .
  .  .  .  . 17  7  6  3
  .  .  .  .  .  .  5  4

Problem:
 . 4 .
 _ 7 _
 1 _ _

Solution:
 . 4 .
 3 7 5
 1 2 6

Problem:
  1  _  _  .  .  _  _  .  .  _  _  .  .  _  _  .  .  _  _  .  .  _  _  .  .  _  _  .  .  _  _  .  .  _  _  .  .  _  _  .  .  _  _  .  .  _  _  .  . 74
  .  .  _  .  _  .  _  .  _  .  _  .  _  .  _  .  _  .  _  .  _  .  _  .  _  .  _  .  _  .  _  .  _  .  _  .  _  .  _  .  _  .  _  .  _  .  _  .  _  .
  .  .  .  _  _  .  .  _  _  .  .  _  _  .  .  _  _  .  .  _  _  .  .  _  _  .  .  _  _  .  .  _  _  .  .  _  _  .  .  _  _  .  .  _  _  .  .  _  _  .

Solution:
  1  2  3  .  .  8  9  .  . 14 15  .  . 20 21  .  . 26 27  .  . 32 33  .  . 38 39  .  . 44 45  .  . 50 51  .  . 56 57  .  . 62 63  .  . 68 69  .  . 74
  .  .  4  .  7  . 10  . 13  . 16  . 19  . 22  . 25  . 28  . 31  . 34  . 37  . 40  . 43  . 46  . 49  . 52  . 55  . 58  . 61  . 64  . 67  . 70  . 73  .
  .  .  .  5  6  .  . 11 12  .  . 17 18  .  . 23 24  .  . 29 30  .  . 35 36  .  . 41 42  .  . 47 48  .  . 53 54  .  . 59 60  .  . 65 66  .  . 71 72  .

Elixir

Translation of: Ruby

<lang elixir># Solve a Hidato Like Puzzle with Warnsdorff like logic applied

defmodule HLPsolver do

 defmodule Cell do
   defstruct value: -1, used: false, adj: []
 end
 
 def solve(str, adjacent, print_out\\true) do
   board = setup(str)
   if print_out, do: print(board, "Problem:")
   {start, _} = Enum.find(board, fn {_,cell} -> cell.value==1 end)
   board = set_adj(board, adjacent)
   zbl = for %Cell{value: n} <- Map.values(board), into: %{}, do: {n, true}
   try do
     solve(board, start, 1, zbl, map_size(board))
     IO.puts "No solution"
   catch
     {:ok, result} -> if print_out, do: print(result, "Solution:"),
                                  else: result
   end
 end
 
 defp solve(board, position, seq_num, zbl, goal) do
   value = board[position].value
   cond do
     value > 0 and value != seq_num -> nil
     value == 0 and zbl[seq_num] -> nil
     true ->
       cell = %Cell{board[position] | value: seq_num, used: true}
       board = %{board | position => cell}
       if seq_num == goal, do: throw({:ok, board})
       Enum.each(wdof(board, cell.adj), fn pos ->
         solve(board, pos, seq_num+1, zbl, goal)
       end)
   end
 end
 
 defp setup(str) do
   lines = String.strip(str) |> String.split(~r/(\n|\r\n|\r)/) |> Enum.with_index
   for {line,i} <- lines, {char,j} <- Enum.with_index(String.split(line)),
       :error != Integer.parse(char), into: %{} do
         {n,_} = Integer.parse(char)
         {{i,j}, %Cell{value: n}}
       end
 end
 
 defp set_adj(board, adjacent) do
   Enum.reduce(Map.keys(board), board, fn {x,y},map ->
     adj = Enum.map(adjacent, fn {i,j} -> {x+i, y+j} end)
           |> Enum.reduce([], fn pos,acc -> if board[pos], do: [pos | acc], else: acc end)
     Map.update!(map, {x,y}, fn cell -> %Cell{cell | adj: adj} end)
   end)
 end
 
 defp wdof(board, adj) do              # Warnsdorf's rule
   Enum.reject(adj, fn pos -> board[pos].used end)
   |> Enum.sort_by(fn pos ->
        Enum.count(board[pos].adj, fn p -> not board[p].used end)
      end)
 end
 
 def print(board, title) do
   IO.puts "\n#{title}"
   {xmin, xmax} = Map.keys(board) |> Enum.map(fn {x,_} -> x end) |> Enum.min_max
   {ymin, ymax} = Map.keys(board) |> Enum.map(fn {_,y} -> y end) |> Enum.min_max
   len = map_size(board) |> to_char_list |> length
   space = String.duplicate(" ", len)
   Enum.each(xmin..xmax, fn x ->
     Enum.map_join(ymin..ymax, " ", fn y ->
       case Map.get(board, {x,y}) do
         nil  -> space
         cell -> to_string(cell.value) |> String.rjust(len)
       end
     end)
     |> IO.puts
   end)
 end

end</lang>

Test: <lang elixir>adjacent = [{-1, -1}, {-1, 0}, {-1, 1}, {0, -1}, {0, 1}, {1, -1}, {1, 0}, {1, 1}]

"""

 .  4
 0  7  0
 1  0  0

""" |> HLPsolver.solve(adjacent)

"""

 0 33 35  0  0
 0  0 24 22  0
 0  0  0 21  0  0
 0 26  0 13 40 11
27  0  0  0  9  0  1
 .  .  0  0 18  0  0
 .  .  .  .  0  7  0  0
 .  .  .  .  .  .  5  0

""" |> HLPsolver.solve(adjacent)

"""

 1  0  0  .  0  0  0  .  0  0  0  .  0  0  0  .  0  0  0  .  0  0  0  .  0  0  0
 .  .  0  .  0  .  0  .  0  .  0  .  0  .  0  .  0  .  0  .  0  .  0  .  0  .  0
 .  .  0  0  0  .  0  0  0  .  0  0  0  .  0  0  0  .  0  0  0  .  0  0  0  .  0

""" |> HLPsolver.solve(adjacent)</lang>

Output:
Problem:
  4  
0 7 0
1 0 0

Solution:
  4  
3 7 5
1 2 6

Problem:
 0 33 35  0  0         
 0  0 24 22  0         
 0  0  0 21  0  0      
 0 26  0 13 40 11      
27  0  0  0  9  0  1   
       0  0 18  0  0   
             0  7  0  0
                   5  0

Solution:
32 33 35 36 37         
31 34 24 22 38         
30 25 23 21 12 39      
29 26 20 13 40 11      
27 28 14 19  9 10  1   
      15 16 18  8  2   
            17  7  6  3
                   5  4

Problem:
 1  0  0     0  0  0     0  0  0     0  0  0     0  0  0     0  0  0     0  0  0
       0     0     0     0     0     0     0     0     0     0     0     0     0
       0  0  0     0  0  0     0  0  0     0  0  0     0  0  0     0  0  0     0

Solution:
 1  2  3     9 10 11    17 18 19    25 26 27    33 34 35    41 42 43    49 50 51
       4     8    12    16    20    24    28    32    36    40    44    48    52
       5  6  7    13 14 15    21 22 23    29 30 31    37 38 39    45 46 47    53

Erlang

To simplify the code I start a new process for searching each potential path through the grid. This means that the default maximum number of processes had to be raised ("erl +P 50000" works for me). The task takes about 1-2 seconds on a low level Mac mini. If faster times are needed, or even less performing hardware is used, some optimisation should be done. <lang Erlang> -module( solve_hidato_puzzle ).

-export( [create/2, solve/1, task/0] ).

-compile({no_auto_import,[max/2]}).

create( Grid_list, Number_list ) ->

       Squares = lists:flatten( [create_column(X, Y) || {X, Y} <- Grid_list] ),

lists:foldl( fun store/2, dict:from_list(Squares), Number_list ).

print( Grid_list ) when is_list(Grid_list) -> print( create(Grid_list, []) ); print( Grid_dict ) ->

   Max_x = max_x( Grid_dict ),
   Max_y = max_y( Grid_dict ),
   Print_row = fun (Y) -> [print(X, Y, Grid_dict) || X <- lists:seq(1, Max_x)], io:nl() end,
   [Print_row(Y) || Y <- lists:seq(1, Max_y)].

solve( Dict ) ->

   {find_start, [Start]} = {find_start, dict:fold( fun start/3, [], Dict )},
   Max = dict:size( Dict ),
   {stop_ok, {Max, Max, [Stop]}} = {stop_ok, dict:fold( fun stop/3, {Max, 0, []}, Dict )},
   My_pid = erlang:self(),
   erlang:spawn( fun() -> path(Start, Stop, Dict, My_pid, []) end ),
   receive
   {grid, Grid, path, Path} -> {Grid, Path}
   end.

task() ->

   %% Square is {X, Y}, N}. N = 0 for empty square. These are created if not present.
   %% Leftmost column is X=1. Top row is Y=1.
   %% Optimised for the example, grid is a list of {X, {Y_min, Y_max}}.
   %% When there are holes, X is repeated as many times as needed with two new Y values each time.
   Start = {{7,5}, 1},
   Stop = {{5,4}, 40},
   Grid_list = [{1, {1,5}}, {2, {1,5}}, {3, {1,6}}, {4, {1,6}}, {5, {1,7}}, {6, {3,7}}, {7, {5,8}}, {8, {7,8}}],
   Number_list = [Start, Stop, {{1,5}, 27}, {{2,1}, 33}, {{2,4}, 26}, {{3,1}, 35}, {{3,2}, 24},
               {{4,2}, 22}, {{4,3}, 21}, {{4,4}, 13}, {{5,5}, 9}, {{5,6}, 18}, {{6,4}, 11}, {{6,7}, 7}, {{7,8}, 5}],
   Grid = create( Grid_list, Number_list ),
   io:fwrite( "Start grid~n" ),
   print( Grid ),
   {New_grid, Path} = solve( create(Grid_list, Number_list) ),
   io:fwrite( "Start square ~p, Stop square ~p.~nPath ~p~n", [Start, Stop, Path] ),
   print( New_grid ).


create_column( X, {Y_min, Y_max} ) -> [{{X, Y}, 0} || Y <- lists:seq(Y_min, Y_max)].

is_filled( Dict ) -> [] =:= dict:fold( fun keep_0_square/3, [], Dict ).

keep_0_square( Key, 0, Acc ) -> [Key | Acc]; keep_0_square( _Key, _Value, Acc ) -> Acc.

max( Position, Keys ) ->

   [Square | _T] = lists:reverse( lists:keysort(Position, Keys) ),
   Square.

max_x( Dict ) ->

   {X, _Y} = max( 1, dict:fetch_keys(Dict) ),
   X.

max_y( Dict ) ->

   {_X, Y} = max( 2, dict:fetch_keys(Dict) ),
   Y.


neighbourhood( Square, Dict ) ->

       Potentials = neighbourhood_potential_squares( Square ),

neighbourhood_squares( dict:find(Square, Dict), Potentials, Dict ).

neighbourhood_potential_squares( {X, Y} ) -> [{Sx, Sy} || Sx <- [X-1, X, X+1], Sy <- [Y-1, Y, Y+1], {X, Y} =/= {Sx, Sy}].

neighbourhood_squares( {ok, Value}, Potentials, Dict ) ->

       Square_values = lists:flatten( [neighbourhood_square_value(X, dict:find(X, Dict)) || X <- Potentials] ),
       Next_value = Value + 1,
       neighbourhood_squares_next_value( lists:keyfind(Next_value, 2, Square_values), Square_values, Next_value ).

neighbourhood_squares_next_value( {Square, Value}, _Square_values, Value ) -> [{Square, Value}]; neighbourhood_squares_next_value( false, Square_values, Value ) -> [{Square, Value} || {Square, Y} <- Square_values, Y =:= 0].

neighbourhood_square_value( Square, {ok, Value} ) -> [{Square, Value}]; neighbourhood_square_value( _Square, error ) -> [].

path( Square, Square, Dict, Pid, Path ) -> path_correct( is_filled(Dict), Pid, [Square | Path], Dict ); path( Square, Stop, Dict, Pid, Path ) ->

   Reversed_path = [Square | Path],
   Neighbours = neighbourhood( Square, Dict ),
   [erlang:spawn( fun() -> path(Next_square, Stop, dict:store(Next_square, Value, Dict), Pid, Reversed_path) end ) || {Next_square, Value} <- Neighbours].

path_correct( true, Pid, Path, Dict ) -> Pid ! {grid, Dict, path, lists:reverse( Path )}; path_correct( false, _Pid, _Path, _Dict ) -> dead_end.

print( X, Y, Dict ) -> print_number( dict:find({X, Y}, Dict) ).

print_number( {ok, 0} ) -> io:fwrite( "~3s", ["."] ); % . is less distracting than 0 print_number( {ok, Value} ) -> io:fwrite( "~3b", [Value] ); print_number( error ) -> io:fwrite( "~3s", [" "] ).

start( Key, 1, Acc ) -> [Key | Acc]; % Allow check that we only have one key with value 1. start( _Key, _Value, Acc ) -> Acc.

stop( Key, Max, {Max, Max_found, Stops} ) -> {Max, erlang:max(Max, Max_found), [Key | Stops]}; % Allow check that we only have one key with value Max. stop( _Key, Value, {Max, Max_found, Stops} ) -> {Max, erlang:max(Value, Max_found), Stops}. % Allow check that Max is Max.

store( {Key, Value}, Dict ) -> dict:store( Key, Value, Dict ). </lang>

Output:
2> solve_hidato_puzzle:task().
Start grid
  . 33 35  .  .         
  .  . 24 22  .         
  .  .  . 21  .  .      
  . 26  . 13 40 11      
 27  .  .  .  9  .  1   
        .  . 18  .  .   
              .  7  .  .
                    5  .
Start square {{7,5},1}, Stop square {{5,4},40}.
Path [{7,5}, {7,6}, {8,7}, {8,8}, {7,8}, {7,7}, {6,7}, {6,6}, {5,5}, {6,5}, {6,4}, {5,3}, {4,4}, {3,5}, {3,6}, {4,6}, {5,7}, {5,6}, {4,5}, {3,4},
      {4,3}, {4,2}, {3,3}, {3,2}, {2,3}, {2,4}, {1,5},{2,5}, {1,4}, {1,3}, {1,2}, {1,1}, {2,1}, {2,2}, {3,1}, {4,1}, {5,1}, {5,2}, {6,3}, {5,4}]
 32 33 35 36 37         
 31 34 24 22 38         
 30 25 23 21 12 39      
 29 26 20 13 40 11      
 27 28 14 19  9 10  1   
       15 16 18  8  2   
             17  7  6  3
                    5  4

Go

Translation of: Java

<lang go>package main

import (

   "fmt"
   "sort"
   "strconv"
   "strings"

)

var board [][]int var start, given []int

func setup(input []string) {

   /* This task is not about input validation, so
      we're going to trust the input to be valid */
   puzzle := make([][]string, len(input))
   for i := 0; i < len(input); i++ {
       puzzle[i] = strings.Fields(input[i])
   }
   nCols := len(puzzle[0])
   nRows := len(puzzle)
   list := make([]int, nRows*nCols)
   board = make([][]int, nRows+2)
   for i := 0; i < nRows+2; i++ {
       board[i] = make([]int, nCols+2)
       for j := 0; j < nCols+2; j++ {
           board[i][j] = -1
       }
   }
   for r := 0; r < nRows; r++ {
       row := puzzle[r]
       for c := 0; c < nCols; c++ {
           switch cell := row[c]; cell {
           case "_":
               board[r+1][c+1] = 0
           case ".":
               break
           default:
               val, _ := strconv.Atoi(cell)
               board[r+1][c+1] = val
               list = append(list, val)
               if val == 1 {
                   start = append(start, r+1, c+1)
               }
           }
       }
   }
   sort.Ints(list)
   given = make([]int, len(list))
   for i := 0; i < len(given); i++ {
       given[i] = list[i]
   }

}

func solve(r, c, n, next int) bool {

   if n > given[len(given)-1] {
       return true
   }
   back := board[r][c]
   if back != 0 && back != n {
       return false
   }
   if back == 0 && given[next] == n {
       return false
   }
   if back == n {
       next++
   }
   board[r][c] = n
   for i := -1; i < 2; i++ {
       for j := -1; j < 2; j++ {
           if solve(r+i, c+j, n+1, next) {
               return true
           }
       }
   }
   board[r][c] = back
   return false

}

func printBoard() {

   for _, row := range board {
       for _, c := range row {
           switch {
           case c == -1:
               fmt.Print(" . ")
           case c > 0:
               fmt.Printf("%2d ", c)
           default:
               fmt.Print("__ ")
           }
       }
       fmt.Println()
   }

}

func main() {

   input := []string{
       "_ 33 35 _ _ . . .",
       "_ _ 24 22 _ . . .",
       "_ _ _ 21 _ _ . .",
       "_ 26 _ 13 40 11 . .",
       "27 _ _ _ 9 _ 1 .",
       ". . _ _ 18 _ _ .",
       ". . . . _ 7 _ _",
       ". . . . . . 5 _",
   }
   setup(input)
   printBoard()
   fmt.Println("\nFound:")
   solve(start[0], start[1], 1, 0)
   printBoard()

}</lang>

Output:
 .  .  .  .  .  .  .  .  .  . 
 . __ 33 35 __ __  .  .  .  . 
 . __ __ 24 22 __  .  .  .  . 
 . __ __ __ 21 __ __  .  .  . 
 . __ 26 __ 13 40 11  .  .  . 
 . 27 __ __ __  9 __  1  .  . 
 .  .  . __ __ 18 __ __  .  . 
 .  .  .  .  . __  7 __ __  . 
 .  .  .  .  .  .  .  5 __  . 
 .  .  .  .  .  .  .  .  .  . 

Found:
 .  .  .  .  .  .  .  .  .  . 
 . 32 33 35 36 37  .  .  .  . 
 . 31 34 24 22 38  .  .  .  . 
 . 30 25 23 21 12 39  .  .  . 
 . 29 26 20 13 40 11  .  .  . 
 . 27 28 14 19  9 10  1  .  . 
 .  .  . 15 16 18  8  2  .  . 
 .  .  .  .  . 17  7  6  3  . 
 .  .  .  .  .  .  .  5  4  . 
 .  .  .  .  .  .  .  .  .  . 

Haskell

<lang haskell>{-# LANGUAGE TupleSections #-} {-# LANGUAGE Rank2Types #-}

import qualified Data.IntMap as I import Data.IntMap (IntMap) import Data.List import Data.Maybe import Data.Time.Clock

data BoardProblem = Board

 { cells :: IntMap (IntMap Int)
 , endVal :: Int
 , onePos :: (Int, Int)
 , givens :: [Int]
 } deriving (Show, Eq)

tupIns x y v m = I.insert x (I.insert y v (I.findWithDefault I.empty x m)) m

tupLookup x y m = I.lookup x m >>= I.lookup y

makeBoard =

 (\x ->
     x
     { givens = dropWhile (<= 1) $ sort $ givens x
     }) .
 foldl' --'
   f
   (Board I.empty 0 (0, 0) []) .
 concatMap (zip [0 ..]) . zipWith (\y w -> map (y, ) $ words w) [0 ..]
 where
   f bd (x, (y, v)) =
     if v == "."
       then bd
       else Board
              (tupIns x y (read v) (cells bd))
              (if read v > endVal bd
                 then read v
                 else endVal bd)
              (if v == "1"
                 then (x, y)
                 else onePos bd)
              (read v : givens bd)

hidato brd = listToMaybe $ h 2 (cells brd) (onePos brd) (givens brd)

 where
   h nval pmap (x, y) gs
     | nval == endVal brd = [pmap]
     | nval == head gs =
       if null nvalAdj
         then []
         else h (nval + 1) pmap (fst $ head nvalAdj) (tail gs)
     | not $ null nvalAdj = h (nval + 1) pmap (fst $ head nvalAdj) gs
     | otherwise = hEmptyAdj
     where
       around =
         [ (x - 1, y - 1)
         , (x, y - 1)
         , (x + 1, y - 1)
         , (x - 1, y)
         , (x + 1, y)
         , (x - 1, y + 1)
         , (x, y + 1)
         , (x + 1, y + 1)
         ]
       lkdUp = map (\(x, y) -> ((x, y), tupLookup x y pmap)) around
       nvalAdj = filter ((== Just nval) . snd) lkdUp
       hEmptyAdj =
         concatMap
           (\((nx, ny), _) -> h (nval + 1) (tupIns nx ny nval pmap) (nx, ny) gs) $
         filter ((== Just 0) . snd) lkdUp

printCellMap cellmap = putStrLn $ concat strings

 where
   maxPos = xyBy I.findMax maximum
   minPos = xyBy I.findMin minimum
   xyBy :: (forall a. IntMap a -> (Int, a)) -> ([Int] -> Int) -> (Int, Int)
   xyBy a b = (fst (a cellmap), b $ map (fst . a . snd) $ I.toList cellmap)
   strings =
     map
       f
       [ (x, y)
       | y <- [snd minPos .. snd maxPos] 
       , x <- [fst minPos .. fst maxPos] ]
   f (x, y) =
     let z =
           if x == fst maxPos
             then "\n"
             else " "
     in case tupLookup x y cellmap of
          Nothing -> "  " ++ z
          Just n ->
            (if n < 10
               then ' ' : show n
               else show n) ++
            z

main = do

 let sampleBoard = makeBoard sample
 printCellMap $ cells sampleBoard
 printCellMap $ fromJust $ hidato sampleBoard

sample =

 [ " 0 33 35  0  0"
 , " 0  0 24 22  0"
 , " 0  0  0 21  0  0"
 , " 0 26  0 13 40 11"
 , "27  0  0  0  9  0  1"
 , ".  .   0  0 18  0  0"
 , ".  .  .  .   0  7  0  0"
 , ".  .  .  .  .  .   5  0"
 ]</lang>
Output:
 0 33 35  0  0         
 0  0 24 22  0         
 0  0  0 21  0  0      
 0 26  0 13 40 11      
27  0  0  0  9  0  1   
       0  0 18  0  0   
             0  7  0  0
                   5  0

32 33 35 36 37         
31 34 24 22 38         
30 25 23 21 12 39      
29 26 20 13 40 11      
27 28 14 19  9 10  1   
      15 16 18  8  2   
            17  7  6  3
                   5  4

Icon and Unicon

This is an Unicon-specific solution but could easily be adjusted to work in Icon. <lang unicon>global nCells, cMap, best record Pos(r,c)

procedure main(A)

   puzzle := showPuzzle("Input",readPuzzle())
   QMouse(puzzle,findStart(puzzle),&null,0)
   showPuzzle("Output", solvePuzzle(puzzle)) | write("No solution!")

end

procedure readPuzzle()

   # Start with a reduced puzzle space
   p := -1
   nCells := maxCols := 0
   every line := !&input do {
       put(p,[: -1 | gencells(line) | -1 :])
       maxCols <:= *p[-1]
       }
   put(p, [-1])
   # Now normalize all rows to the same length
   every i := 1 to *p do p[i] := [: !p[i] | (|-1\(maxCols - *p[i])) :]
   return p

end

procedure gencells(s)

   static WS, NWS
   initial {
       NWS := ~(WS := " \t")
       cMap := table()     # Map to/from internal model
       cMap["#"] := -1;  cMap["_"] :=  0
       cMap[-1]  := " "; cMap[0]   := "_"
       }
   s ? while not pos(0) do {
           w := (tab(many(WS))|"", tab(many(NWS))) | break
           w := numeric(\cMap[w]|w)
           if -1 ~= w then nCells +:= 1
           suspend w
           }

end

procedure showPuzzle(label, p)

   write(label," with ",nCells," cells:")
   every r := !p do {
       every c := !r do writes(right((\cMap[c]|c),*nCells+1))
       write()
       }
   return p

end

procedure findStart(p)

   if \p[r := !*p][c := !*p[r]] = 1 then return Pos(r,c)

end

procedure solvePuzzle(puzzle)

   if path := \best then {
       repeat {
           loc := path.getLoc()
           puzzle[loc.r][loc.c] := path.getVal()
           path := \path.getParent() | break
           }
       return puzzle
       }

end

class QMouse(puzzle, loc, parent, val)

   method getVal(); return val; end
   method getLoc(); return loc; end
   method getParent(); return parent; end
   method atEnd(); return (nCells = val) = puzzle[loc.r][loc.c]; end
   method goNorth(); return visit(loc.r-1,loc.c);   end
   method goNE();    return visit(loc.r-1,loc.c+1); end
   method goEast();  return visit(loc.r,  loc.c+1); end
   method goSE();    return visit(loc.r+1,loc.c+1); end
   method goSouth(); return visit(loc.r+1,loc.c);   end
   method goSW();    return visit(loc.r+1,loc.c-1); end
   method goWest();  return visit(loc.r,  loc.c-1); end
   method goNW();    return visit(loc.r-1,loc.c-1); end
   method visit(r,c)
       if /best & validPos(r,c) then return Pos(r,c)
   end
   method validPos(r,c)
       xv := puzzle[r][c]
       if xv = (val+1) then return
       if xv = 0 then {  # make sure this path hasn't already gone there
           ancestor := self
           while xl := (ancestor := \ancestor.getParent()).getLoc() do
               if (xl.r = r) & (xl.c = c) then fail
           return
           }
   end

initially

   val +:= 1
   if atEnd() then return best := self
   QMouse(puzzle, goNorth(), self, val)
   QMouse(puzzle, goNE(),    self, val)
   QMouse(puzzle, goEast(),  self, val)
   QMouse(puzzle, goSE(),    self, val)
   QMouse(puzzle, goSouth(), self, val)
   QMouse(puzzle, goSW(),    self, val)
   QMouse(puzzle, goWest(),  self, val)
   QMouse(puzzle, goNW(),    self, val)

end</lang>

Sample run:

->hd <hd.in 
Input with 40 cells:
                              
     _ 33 35  _  _            
     _  _ 24 22  _            
     _  _  _ 21  _  _         
     _ 26  _ 13 40 11         
    27  _  _  _  9  _  1      
           _  _ 18  _  _      
                 _  7  _  _   
                       5  _   
                              
Output with 40 cells:
                              
    32 33 35 36 37            
    31 34 24 22 38            
    30 25 23 21 12 39         
    29 26 20 13 40 11         
    27 28 14 19  9 10  1      
          15 16 18  8  2      
                17  7  6  3   
                       5  4   
                              
->

Java

Translation of: D
Works with: Java version 7

<lang java>import java.util.ArrayList; import java.util.Collections; import java.util.List;

public class Hidato {

   private static int[][] board;
   private static int[] given, start;
   public static void main(String[] args) {
       String[] input = {"_ 33 35 _ _ . . .",
           "_ _ 24 22 _ . . .",
           "_ _ _ 21 _ _ . .",
           "_ 26 _ 13 40 11 . .",
           "27 _ _ _ 9 _ 1 .",
           ". . _ _ 18 _ _ .",
           ". . . . _ 7 _ _",
           ". . . . . . 5 _"};
       setup(input);
       printBoard();
       System.out.println("\nFound:");
       solve(start[0], start[1], 1, 0);
       printBoard();
   }
   private static void setup(String[] input) {
       /* This task is not about input validation, so
          we're going to trust the input to be valid */
       String[][] puzzle = new String[input.length][];
       for (int i = 0; i < input.length; i++)
           puzzle[i] = input[i].split(" ");
       int nCols = puzzle[0].length;
       int nRows = puzzle.length;
       List<Integer> list = new ArrayList<>(nRows * nCols);
       board = new int[nRows + 2][nCols + 2];
       for (int[] row : board)
           for (int c = 0; c < nCols + 2; c++)
               row[c] = -1;
       for (int r = 0; r < nRows; r++) {
           String[] row = puzzle[r];
           for (int c = 0; c < nCols; c++) {
               String cell = row[c];
               switch (cell) {
                   case "_":
                       board[r + 1][c + 1] = 0;
                       break;
                   case ".":
                       break;
                   default:
                       int val = Integer.parseInt(cell);
                       board[r + 1][c + 1] = val;
                       list.add(val);
                       if (val == 1)
                           start = new int[]{r + 1, c + 1};
               }
           }
       }
       Collections.sort(list);
       given = new int[list.size()];
       for (int i = 0; i < given.length; i++)
           given[i] = list.get(i);
   }
   private static boolean solve(int r, int c, int n, int next) {
       if (n > given[given.length - 1])
           return true;
       if (board[r][c] != 0 && board[r][c] != n)
           return false;
       if (board[r][c] == 0 && given[next] == n)
           return false;
       int back = board[r][c];
       if (back == n)
           next++;
       board[r][c] = n;
       for (int i = -1; i < 2; i++)
           for (int j = -1; j < 2; j++)
               if (solve(r + i, c + j, n + 1, next))
                   return true;
       board[r][c] = back;
       return false;
   }
   private static void printBoard() {
       for (int[] row : board) {
           for (int c : row) {
               if (c == -1)
                   System.out.print(" . ");
               else
                   System.out.printf(c > 0 ? "%2d " : "__ ", c);
           }
           System.out.println();
       }
   }

}</lang>

Output:

 .  .  .  .  .  .  .  .  .  . 
 . __ 33 35 __ __  .  .  .  . 
 . __ __ 24 22 __  .  .  .  . 
 . __ __ __ 21 __ __  .  .  . 
 . __ 26 __ 13 40 11  .  .  . 
 . 27 __ __ __  9 __  1  .  . 
 .  .  . __ __ 18 __ __  .  . 
 .  .  .  .  . __  7 __ __  . 
 .  .  .  .  .  .  .  5 __  . 
 .  .  .  .  .  .  .  .  .  . 

Found:
 .  .  .  .  .  .  .  .  .  . 
 . 32 33 35 36 37  .  .  .  . 
 . 31 34 24 22 38  .  .  .  . 
 . 30 25 23 21 12 39  .  .  . 
 . 29 26 20 13 40 11  .  .  . 
 . 27 28 14 19  9 10  1  .  . 
 .  .  . 15 16 18  8  2  .  . 
 .  .  .  .  . 17  7  6  3  . 
 .  .  .  .  .  .  .  5  4  . 
 .  .  .  .  .  .  .  .  .  .

Kotlin

Translation of: Java

<lang scala>// version 1.2.0

lateinit var board: List<IntArray> lateinit var given: IntArray lateinit var start: IntArray

fun setUp(input: List<String>) {

   val nRows = input.size
   val puzzle = List(nRows) { input[it].split(" ") }
   val nCols = puzzle[0].size
   val list = mutableListOf<Int>()
   board = List(nRows + 2) { IntArray(nCols + 2) { -1 } }
   for (r in 0 until nRows) {
       val row = puzzle[r]
       for (c in 0 until nCols) {
           val cell = row[c]
           if (cell == "_") {
               board[r + 1][c + 1] = 0
           }
           else if (cell != ".") {
               val value = cell.toInt()
               board[r + 1][c + 1] = value
               list.add(value)
               if (value == 1) start = intArrayOf(r + 1, c + 1)
           }
       }
   }
   list.sort()
   given = list.toIntArray()

}

fun solve(r: Int, c: Int, n: Int, next: Int): Boolean {

   if (n > given[given.lastIndex]) return true
   val back = board[r][c]
   if (back != 0 && back != n) return false
   if (back == 0 && given[next] == n) return false
   var next2 = next
   if (back == n) next2++
   board[r][c] = n
   for (i in -1..1)
       for (j in -1..1)
           if (solve(r + i, c + j, n + 1, next2)) return true
   board[r][c] = back
   return false

}

fun printBoard() {

   for (row in board) {
       for (c in row) {
           if (c == -1)
               print(" . ")
           else
               print(if (c > 0) "%2d ".format(c) else "__ ")
       }
       println()
   }

}

fun main(args: Array<String>) {

   var input = listOf(
       "_ 33 35 _ _ . . .",
       "_ _ 24 22 _ . . .",
       "_ _ _ 21 _ _ . .",
       "_ 26 _ 13 40 11 . .",
       "27 _ _ _ 9 _ 1 .",
       ". . _ _ 18 _ _ .",
       ". . . . _ 7 _ _",
       ". . . . . . 5 _"
   )
   setUp(input)
   printBoard()
   println("\nFound:")
   solve(start[0], start[1], 1, 0)
   printBoard()

}</lang>

Output:
 .  .  .  .  .  .  .  .  .  . 
 . __ 33 35 __ __  .  .  .  . 
 . __ __ 24 22 __  .  .  .  . 
 . __ __ __ 21 __ __  .  .  . 
 . __ 26 __ 13 40 11  .  .  . 
 . 27 __ __ __  9 __  1  .  . 
 .  .  . __ __ 18 __ __  .  . 
 .  .  .  .  . __  7 __ __  . 
 .  .  .  .  .  .  .  5 __  . 
 .  .  .  .  .  .  .  .  .  . 

Found:
 .  .  .  .  .  .  .  .  .  . 
 . 32 33 35 36 37  .  .  .  . 
 . 31 34 24 22 38  .  .  .  . 
 . 30 25 23 21 12 39  .  .  . 
 . 29 26 20 13 40 11  .  .  . 
 . 27 28 14 19  9 10  1  .  . 
 .  .  . 15 16 18  8  2  .  . 
 .  .  .  .  . 17  7  6  3  . 
 .  .  .  .  .  .  .  5  4  . 
 .  .  .  .  .  .  .  .  .  . 

Mathprog

<lang mathprog>/*Hidato.mathprog, part of KuKu by Nigel Galloway

 Find a solution to a Hidato problem
 Nigel_Galloway@operamail.com
 April 1st., 2011
  • /

param ZBLS; param ROWS; param COLS; param D := 1; set ROWSR := 1..ROWS; set COLSR := 1..COLS; set ROWSV := (1-D)..(ROWS+D); set COLSV := (1-D)..(COLS+D); param Iz{ROWSR,COLSR}, integer, default 0; set ZBLSV := 1..(ZBLS+1); set ZBLSR := 1..ZBLS;

var BR{ROWSV,COLSV,ZBLSV}, binary;

void0{r in ROWSV, z in ZBLSR,c in (1-D)..0}: BR[r,c,z] = 0; void1{r in ROWSV, z in ZBLSR,c in (COLS+1)..(COLS+D)}: BR[r,c,z] = 0; void2{c in COLSV, z in ZBLSR,r in (1-D)..0}: BR[r,c,z] = 0; void3{c in COLSV, z in ZBLSR,r in (ROWS+1)..(ROWS+D)}: BR[r,c,z] = 0; void4{r in ROWSV,c in (1-D)..0}: BR[r,c,ZBLS+1] = 1; void5{r in ROWSV,c in (COLS+1)..(COLS+D)}: BR[r,c,ZBLS+1] = 1; void6{c in COLSV,r in (1-D)..0}: BR[r,c,ZBLS+1] = 1; void7{c in COLSV,r in (ROWS+1)..(ROWS+D)}: BR[r,c,ZBLS+1] = 1;

Izfree{r in ROWSR, c in COLSR, z in ZBLSR : Iz[r,c] = -1}: BR[r,c,z] = 0; Iz1{Izr in ROWSR, Izc in COLSR, r in ROWSR, c in COLSR, z in ZBLSR : Izr=r and Izc=c and Iz[Izr,Izc]=z}: BR[r,c,z] = 1;

rule1{z in ZBLSR}: sum{r in ROWSR, c in COLSR} BR[r,c,z] = 1; rule2{r in ROWSR, c in COLSR}: sum{z in ZBLSV} BR[r,c,z] = 1; rule3{r in ROWSR, c in COLSR, z in ZBLSR}: BR[0,0,z+1] + BR[r-1,c-1,z+1] + BR[r-1,c,z+1] + BR[r-1,c+1,z+1] + BR[r,c-1,z+1] + BR[r,c+1,z+1] + BR[r+1,c-1,z+1] + BR[r+1,c,z+1] + BR[r+1,c+1,z+1] - BR[r,c,z] >= 0;

solve;

for {r in ROWSR} {

   for {c in COLSR} {
       printf " %2d", sum{z in ZBLSR} BR[r,c,z]*z;
   }
   printf "\n";

} data;

param ROWS := 8; param COLS := 8; param ZBLS := 40; param Iz: 1 2 3 4 5 6 7 8 :=

1  .  33  35   .   .  -1  -1  -1
2  .   .  24  22   .  -1  -1  -1
3  .   .   .  21   .   .  -1  -1
4  .  26   .  13  40  11  -1  -1
5 27   .   .   .   9   .   1  -1 
6 -1  -1   .   .  18   .   .  -1 
7 -1  -1  -1  -1   .   7   .   .
8 -1  -1  -1  -1  -1  -1   5   . 
;

end;</lang>

Using the data in the model produces the following:

Output:
>glpsol --minisat --math Hidato.mathprog
GLPSOL: GLPK LP/MIP Solver, v4.47
Parameter(s) specified in the command line:
 --minisat --math Hidato.mathprog
Reading model section from Hidato.mathprog...
Reading data section from Hidato.mathprog...
64 lines were read
Generating void0...
Generating void1...
Generating void2...
Generating void3...
Generating void4...
Generating void5...
Generating void6...
Generating void7...
Generating Izfree...
Generating Iz1...
Generating rule1...
Generating rule2...
Generating rule3...
Model has been successfully generated
Will search for ANY feasible solution
Translating to CNF-SAT...
Original problem has 5279 rows, 4100 columns, and 33359 non-zeros
2520 covering inequalities
2719 partitioning equalities
Solving CNF-SAT problem...
Instance has 7076 variables, 24047 clauses, and 77735 literals
==================================[MINISAT]===================================
| Conflicts |     ORIGINAL     |              LEARNT              | Progress |
|           | Clauses Literals |   Limit Clauses Literals  Lit/Cl |          |
==============================================================================
|         0 |   21432    75120 |    7144       0        0     0.0 |  0.000 % |
==============================================================================
SATISFIABLE
Objective value =  0.000000000e+000
Time used:   0.0 secs
Memory used: 14.5 Mb (15192264 bytes)
 32 33 35 36 37  0  0  0
 31 34 24 22 38  0  0  0
 30 25 23 21 12 39  0  0
 29 26 20 13 40 11  0  0
 27 28 14 19  9 10  1  0
  0  0 15 16 18  8  2  0
  0  0  0  0 17  7  6  3
  0  0  0  0  0  0  5  4
Model has been successfully processed

Modelling Evil Case 1:

data;
param ROWS := 3;
param COLS := 3;
param ZBLS := 7;
param
Iz: 1   2   3 :=
 1 -1   4  -1
 2  .   7   .
 3  1   .   .
 ;
end;

Produces:

>glpsol --minisat --math Hidato.mathprog --data Evil1.data
GLPSOL: GLPK LP/MIP Solver, v4.47
Parameter(s) specified in the command line:
 --minisat --math Hidato.mathprog --data Evil1.data
Reading model section from Hidato.mathprog...
Hidato.mathprog:47: warning: data section ignored
47 lines were read
Reading data section from Evil1.data...
11 lines were read
Generating void0...
Generating void1...
Generating void2...
Generating void3...
Generating void4...
Generating void5...
Generating void6...
Generating void7...
Generating Izfree...
Generating Iz1...
Generating rule1...
Generating rule2...
Generating rule3...
Model has been successfully generated
Will search for ANY feasible solution
Translating to CNF-SAT...
Original problem has 256 rows, 200 columns, and 935 non-zeros
56 covering inequalities
193 partitioning equalities
Solving CNF-SAT problem...
Instance has 337 variables, 1237 clauses, and 4094 literals
==================================[MINISAT]===================================
| Conflicts |     ORIGINAL     |              LEARNT              | Progress |
|           | Clauses Literals |   Limit Clauses Literals  Lit/Cl |          |
==============================================================================
|         0 |    1060     3917 |     353       0        0     0.0 |  0.000 % |
==============================================================================
SATISFIABLE
Objective value =  0.000000000e+000
Time used:   0.0 secs
Memory used: 0.8 Mb (861188 bytes)
  0  4  0
  3  7  5
  1  2  6
Model has been successfully processed

Modelling Evil Case 2 - The Snake in the Grass:

data;
param ROWS := 3;
param COLS := 50;
param ZBLS := 74;
param
Iz:  1  2  3  4  5  6  7  8  9 10 11 12 13 14 15 16 17 18 19 20 21 22 23 24 25 26 27 28 29 30 31 32 33 34 35 36 37 38 39 40 41 42 43 44 45 46 47 48 49 50 :=
 1   1  .  . -1 -1  .  . -1 -1  .  . -1 -1  .  . -1 -1  .  . -1 -1  .  . -1 -1  .  . -1 -1  .  . -1 -1  .  . -1 -1  .  . -1 -1  .  . -1 -1  .  . -1 -1 74
 2  -1 -1  . -1  . -1  . -1  . -1  . -1  . -1  . -1  . -1  . -1  . -1  . -1  . -1  . -1  . -1  . -1  . -1  . -1  . -1  . -1  . -1  . -1  . -1  . -1  . -1
 3  -1 -1 -1  .  . -1 -1  .  . -1 -1  .  . -1 -1  .  . -1 -1  .  . -1 -1  .  . -1 -1  .  . -1 -1  .  . -1 -1  .  . -1 -1  .  . -1 -1  .  . -1 -1  .  . -1
;
end;

Produces:

G:\IAJAAR4.47>glpsol --minisat --math Hidato.mathprog --data Evil2.data
GLPSOL: GLPK LP/MIP Solver, v4.47
Parameter(s) specified in the command line:
 --minisat --math Hidato.mathprog --data Evil2.data
Reading model section from Hidato.mathprog...
Hidato.mathprog:47: warning: data section ignored
47 lines were read
Reading data section from Evil2.data...
Evil2.data:11: warning: final NL missing before end of file
11 lines were read
Generating void0...
Generating void1...
Generating void2...
Generating void3...
Generating void4...
Generating void5...
Generating void6...
Generating void7...
Generating Izfree...
Generating Iz1...
Generating rule1...
Generating rule2...
Generating rule3...
Model has been successfully generated
Will search for ANY feasible solution
Translating to CNF-SAT...
Original problem has 25500 rows, 19500 columns, and 147452 non-zeros
11026 covering inequalities
14400 partitioning equalities
Solving CNF-SAT problem...
Instance has 31338 variables, 98310 clauses, and 305726 literals
==================================[MINISAT]===================================
| Conflicts |     ORIGINAL     |              LEARNT              | Progress |
|           | Clauses Literals |   Limit Clauses Literals  Lit/Cl |          |
==============================================================================
|         0 |   84134   291550 |   28044       0        0     0.0 |  0.000 % |
|       101 |   31135   126809 |   30848      98     5496    56.1 | 65.521 % |
|       251 |   31135   126809 |   33933     244    12470    51.1 | 66.552 % |
|       476 |   27353   115512 |   37327     446    23819    53.4 | 68.160 % |
|       814 |   26574   113330 |   41059     770    42161    54.8 | 69.586 % |
|      1321 |   25432   110534 |   45165    1262    83658    66.3 | 70.056 % |
==============================================================================
SATISFIABLE
Objective value =  0.000000000e+000
Time used:   1.0 secs
Memory used: 60.9 Mb (63862624 bytes)
  1  2  3  0  0  8  9  0  0 14 15  0  0 20 21  0  0 26 27  0  0 32 33  0  0 38 39  0  0 44 45  0  0 50 51  0  0 56 57  0  0 62 63  0  0 68 69  0  0 74
  0  0  4  0  7  0 10  0 13  0 16  0 19  0 22  0 25  0 28  0 31  0 34  0 37  0 40  0 43  0 46  0 49  0 52  0 55  0 58  0 61  0 64  0 67  0 70  0 73  0
  0  0  0  5  6  0  0 11 12  0  0 17 18  0  0 23 24  0  0 29 30  0  0 35 36  0  0 41 42  0  0 47 48  0  0 53 54  0  0 59 60  0  0 65 66  0  0 71 72  0
Model has been successfully processed

Modelling Evil Case 3 - A fatter snake in the Grass:

data;
param ROWS := 4;
param COLS := 46;
param ZBLS := 82;
param
Iz:  1  2  3  4  5  6  7  8  9 10 11 12 13 14 15 16 17 18 19 20 21 22 23 24 25 26 27 28 29 30 31 32 33 34 35 36 37 38 39 40 41 42 43 44 45 46 :=
 1   1  0 -1 -1 -1  0  0 -1 -1 -1  0  0 -1 -1 -1  0  0 -1 -1 -1  0  0 -1 -1 -1  0  0 -1 -1 -1  0  0 -1 -1 -1  0  0 -1 -1 -1  0  0 -1 -1 -1 82
 2  -1 -1  0 -1  0 -1 -1  0 -1  0 -1 -1  0 -1  0 -1 -1  0 -1  0 -1 -1  0 -1  0 -1 -1  0 -1  0 -1 -1  0 -1  0 -1 -1  0 -1  0 -1 -1  0 -1  0 -1 
 3  -1  0 -1  0 -1 -1  0 -1  0 -1 -1  0 -1  0 -1 -1  0 -1  0 -1 -1  0 -1  0 -1 -1  0 -1  0 -1 -1  0 -1  0 -1 -1  0 -1  0 -1 -1  0 -1  0 -1 -1 
 4   0  0  0 -1 -1  0  0  0 -1 -1  0  0  0 -1 -1  0  0  0 -1 -1  0  0  0 -1 -1  0  0  0 -1 -1  0  0  0 -1 -1  0  0  0 -1 -1  0  0  0 -1 -1 -1
 ;
end;

Produces:

>glpsol --minisat --math Hidato.mathprog --data Evil3.data
GLPSOL: GLPK LP/MIP Solver, v4.47
Parameter(s) specified in the command line:
 --minisat --math Hidato.mathprog --data Evil3.data
Reading model section from Hidato.mathprog...
Hidato.mathprog:47: warning: data section ignored
47 lines were read
Reading data section from Evil3.data...
12 lines were read
Generating void0...
Generating void1...
Generating void2...
Generating void3...
Generating void4...
Generating void5...
Generating void6...
Generating void7...
Generating Izfree...
Generating Iz1...
Generating rule1...
Generating rule2...
Generating rule3...
Model has been successfully generated
Will search for ANY feasible solution
Translating to CNF-SAT...
Original problem has 32684 rows, 23904 columns, and 198488 non-zeros
15006 covering inequalities
17596 partitioning equalities
Solving CNF-SAT problem...
Instance has 39792 variables, 130040 clauses, and 407222 literals
==================================[MINISAT]===================================
| Conflicts |     ORIGINAL     |              LEARNT              | Progress |
|           | Clauses Literals |   Limit Clauses Literals  Lit/Cl |          |
==============================================================================
|         0 |  112710   389892 |   37570       0        0     0.0 |  0.000 % |
==============================================================================
SATISFIABLE
Objective value =  0.000000000e+000
Time used:   0.0 secs
Memory used: 80.2 Mb (84067912 bytes)
  1  2  0  0  0 10 11  0  0  0 19 20  0  0  0 28 29  0  0  0 37 38  0  0  0 46 47  0  0  0 55 56  0  0  0 64 65  0  0  0 73 74  0  0  0 82
  0  0  3  0  9  0  0 12  0 18  0  0 21  0 27  0  0 30  0 36  0  0 39  0 45  0  0 48  0 54  0  0 57  0 63  0  0 66  0 72  0  0 75  0 81  0
  0  4  0  8  0  0 13  0 17  0  0 22  0 26  0  0 31  0 35  0  0 40  0 44  0  0 49  0 53  0  0 58  0 62  0  0 67  0 71  0  0 76  0 80  0  0
  5  6  7  0  0 14 15 16  0  0 23 24 25  0  0 32 33 34  0  0 41 42 43  0  0 50 51 52  0  0 59 60 61  0  0 68 69 70  0  0 77 78 79  0  0  0
Model has been successfully processed

Nim

<lang nim>import strutils, algorithm

var board: array[0..19, array[0..19, int]] var given, start: seq[int] = @[] var rows, cols: int = 0

proc setup(s: string) =

   var lines = s.splitLines()
   cols = lines[0].split().len()
   rows = lines.len()
   for i in 0 .. rows + 1:
       for j in 0 .. cols + 1:
          board[i][j] = -1

   for r, row in pairs(lines):
       for c, cell in pairs(row.split()):
           case cell
           of "__" : 
               board[r + 1][c + 1] = 0
               continue
           of "." : continue
           else :
              var val = parseInt(cell)
              board[r + 1][c + 1] = val
              given.add(val)
              if (val == 1): 
                  start.add(r + 1)
                  start.add(c + 1)
   given.sort(cmp[int], Ascending)

proc solve(r, c, n: int, next: int = 0): bool =

   if n > given[high(given)]:
      return true
   if board[r][c] < 0:
       return false
   if (board[r][c] > 0 and board[r][c] != n):
       return false
   if (board[r][c] == 0 and given[next] == n):
       return false
   var back = board[r][c]
   board[r][c] = n
   for i in -1 .. 1:
       for j in -1 .. 1:
           if back == n:
               if (solve(r + i, c + j, n + 1, next + 1)):  return true
           else:
               if (solve(r + i, c + j, n + 1, next)): return true
   board[r][c] = back
   result = false


proc printBoard() =

   for r in  0 .. rows + 1:
       for cellid,c in pairs(board[r]):
           if cellid > cols + 1: break
           if c == -1:
               write(stdout, " . ")
           elif c == 0:
               write(stdout, "__ ")
           else:
               write(stdout, "$# " % align($c,2))
       writeLine(stdout, "")

var hi: string = """__ 33 35 __ __ . . . __ __ 24 22 __ . . . __ __ __ 21 __ __ . . __ 26 __ 13 40 11 . . 27 __ __ __ 9 __ 1 . . . __ __ 18 __ __ . . . . . __ 7 __ __ . . . . . . 5 __"""

setup(hi) printBoard() echo("") echo("Found:") discard solve(start[0], start[1], 1) printBoard()</lang>

Output:
 .  .  .  .  .  .  .  .  .  . 
 . __ 33 35 __ __  .  .  .  . 
 . __ __ 24 22 __  .  .  .  . 
 . __ __ __ 21 __ __  .  .  . 
 . __ 26 __ 13 40 11  .  .  . 
 . 27 __ __ __  9 __  1  .  . 
 .  .  . __ __ 18 __ __  .  . 
 .  .  .  .  . __  7 __ __  . 
 .  .  .  .  .  .  .  5 __  . 
 .  .  .  .  .  .  .  .  .  . 

Found:
 .  .  .  .  .  .  .  .  .  . 
 . 32 33 35 36 37  .  .  .  . 
 . 31 34 24 22 38  .  .  .  . 
 . 30 25 23 21 12 39  .  .  . 
 . 29 26 20 13 40 11  .  .  . 
 . 27 28 14 19  9 10  1  .  . 
 .  .  . 15 16 18  8  2  .  . 
 .  .  .  .  . 17  7  6  3  . 
 .  .  .  .  .  .  .  5  4  . 
 .  .  .  .  .  .  .  .  .  . 

Perl

<lang perl>use strict; use List::Util 'max';

our (@grid, @known, $n);

sub show_board { for my $r (@grid) { print map(!defined($_) ? ' ' : $_ ? sprintf("%3d", $_) : ' __' , @$r), "\n" } }

sub parse_board { @grid = map{[map(/^_/ ? 0 : /^\./ ? undef: $_, split ' ')]} split "\n", shift(); for my $y (0 .. $#grid) { for my $x (0 .. $#{$grid[$y]}) { $grid[$y][$x] > 0 and $known[$grid[$y][$x]] = "$y,$x"; } } $n = max(map { max @$_ } @grid); }

sub neighbors { my ($y, $x) = @_; my @out; for ( [-1, -1], [-1, 0], [-1, 1], [ 0, -1], [ 0, 1], [ 1, -1], [ 1, 0], [ 1, 1]) { my $y1 = $y + $_->[0]; my $x1 = $x + $_->[1]; next if $x1 < 0 || $y1 < 0; next unless defined $grid[$y1][$x1]; push @out, "$y1,$x1"; } @out }

sub try_fill { my ($v, $coord) = @_; return 1 if $v > $n;

my ($y, $x) = split ',', $coord; my $old = $grid[$y][$x];

return if $old && $old != $v; return if exists $known[$v] and $known[$v] ne $coord;

$grid[$y][$x] = $v; print "\033[0H"; show_board();

try_fill($v + 1, $_) && return 1 for neighbors($y, $x);

$grid[$y][$x] = $old; return }

parse_board

  1. ". 4 .
  2. _ 7 _
  3. 1 _ _";
  1. " 1 _ _ . . _ _ . . _ _ . . _ _ . . _ _ . . _ _ . . _ _ . . _ _ . . _ _ . . _ _ . . _ _ . . _ _ . . 74
  2. . . _ . _ . _ . _ . _ . _ . _ . _ . _ . _ . _ . _ . _ . _ . _ . _ . _ . _ . _ . _ . _ . _ . _ . _
  3. . . . _ _ . . _ _ . . _ _ . . _ _ . . _ _ . . _ _ . . _ _ . . _ _ . . _ _ . . _ _ . . _ _ . . _ _
  4. ";

"__ 33 35 __ __ .. .. .. . __ __ 24 22 __ .. .. .. . __ __ __ 21 __ __ .. .. . __ 26 __ 13 40 11 .. .. . 27 __ __ __ 9 __ 1 .. . . . __ __ 18 __ __ .. . . .. . . __ 7 __ __ . . .. .. .. . . 5 __ .";

print "\033[2J";

try_fill(1, $known[1]);</lang>

Output:
32 33 35 36 37            
31 34 24 22 38            
30 25 23 21 12 39         
29 26 20 13 40 11         
27 28 14 19  9 10  1      
      15 16 18  8  2      
            17  7  6  3   
                   5  4

Perl 6

This uses a Warnsdorff solver, which cuts down the number of tries by more than a factor of six over the brute force approach. This same solver is used in:

<lang perl6>my @adjacent = [-1, -1], [-1, 0], [-1, 1],

              [ 0, -1],          [ 0, 1],
              [ 1, -1], [ 1, 0], [ 1, 1];

solveboard q:to/END/;

       __ 33 35 __ __ .. .. ..
       __ __ 24 22 __ .. .. ..
       __ __ __ 21 __ __ .. ..
       __ 26 __ 13 40 11 .. ..
       27 __ __ __  9 __  1 ..
       .. .. __ __ 18 __ __ ..
       .. .. .. .. __  7 __ __
       .. .. .. .. .. ..  5 __
       END

sub solveboard($board) {

   my $max = +$board.comb(/\w+/);
   my $width = $max.chars;
   my @grid;
   my @known;
   my @neigh;
   my @degree;

   @grid = $board.lines.map: -> $line {
       [ $line.words.map: { /^_/ ?? 0 !! /^\./ ?? Rat !! $_ } ]
   }

   sub neighbors($y,$x --> List) {
       eager gather for @adjacent {
           my $y1 = $y + .[0];
           my $x1 = $x + .[1];
           take [$y1,$x1] if defined @grid[$y1][$x1];
       }
   }
   for ^@grid -> $y {
       for ^@grid[$y] -> $x {
           if @grid[$y][$x] -> $v {
               @known[$v] = [$y,$x];
           }
           if @grid[$y][$x].defined {
               @neigh[$y][$x] = neighbors($y,$x);
               @degree[$y][$x] = +@neigh[$y][$x];
           }
       }
   }
   print "\e[0H\e[0J";
   my $tries = 0;
   try_fill 1, @known[1];
   sub try_fill($v, $coord [$y,$x] --> Bool) {
       return True if $v > $max;
       $tries++;
       my $old = @grid[$y][$x];
       return False if +$old and $old != $v;
       return False if @known[$v] and @known[$v] !eqv $coord;
       @grid[$y][$x] = $v;               # conjecture grid value
       print "\e[0H";                    # show conjectured board
       for @grid -> $r {
           say do for @$r {
               when Rat { ' ' x $width }
               when 0   { '_' x $width }
               default  { .fmt("%{$width}d") }
           }
       }


       my @neighbors = @neigh[$y][$x][];
       my @degrees;
       for @neighbors -> \n [$yy,$xx] {
           my $d = --@degree[$yy][$xx];  # conjecture new degrees
           push @degrees[$d], n;         # and categorize by degree
       }
       for @degrees.grep(*.defined) -> @ties {
           for @ties.reverse {           # reverse works better for this hidato anyway
               return True if try_fill $v + 1, $_;
           }
       }
       for @neighbors -> [$yy,$xx] {
           ++@degree[$yy][$xx];          # undo degree conjectures
       }
       @grid[$y][$x] = $old;             # undo grid value conjecture
       return False;
   }
    
   say "$tries tries";

}</lang>

Phix

<lang Phix>sequence board, warnsdorffs, knownx, knowny

integer width, height, limit, nchars, tries string fmt, blank

constant ROW = 1, COL = 2 constant moves = {{-1,-1},{-1,0},{-1,1},{0,-1},{0,1},{1,-1},{1,0},{1,1}}

function onboard(integer row, integer col)

   return row>=1 and row<=height and col>=nchars and col<=nchars*width

end function

procedure init_warnsdorffs() integer nrow,ncol

   for row=1 to height do
       for col=nchars to nchars*width by nchars do
           for move=1 to length(moves) do
               nrow = row+moves[move][ROW]
               ncol = col+moves[move][COL]*nchars
               if onboard(nrow,ncol)
               and board[nrow][ncol]='_' then
                   warnsdorffs[nrow][ncol] += 1
               end if
           end for
       end for
   end for

end procedure

function solve(integer row, integer col, integer n) integer nrow, ncol

   tries+= 1
   if n>limit then return 1 end if
   if knownx[n] then
       for move=1 to length(moves) do
           nrow = row+moves[move][ROW]
           ncol = col+moves[move][COL]*nchars
           if nrow = knownx[n]
           and ncol = knowny[n] then
               if solve(nrow,ncol,n+1) then return 1 end if
               exit
           end if
       end for
       return 0
   end if
   sequence wmoves = {}
   for move=1 to length(moves) do
       nrow = row+moves[move][ROW]
       ncol = col+moves[move][COL]*nchars
       if onboard(nrow,ncol)
       and board[nrow][ncol]='_' then
           wmoves = append(wmoves,{warnsdorffs[nrow][ncol],nrow,ncol})
       end if
   end for
   wmoves = sort(wmoves)
   -- avoid creating orphans
   if length(wmoves)<2 or wmoves[2][1]>1 then
       for m=1 to length(wmoves) do
           {?,nrow,ncol} = wmoves[m]
           warnsdorffs[nrow][ncol] -= 1
       end for
       for m=1 to length(wmoves) do
           {?,nrow,ncol} = wmoves[m]
           board[nrow][ncol-nchars+1..ncol] = sprintf(fmt,n)
           if solve(nrow,ncol,n+1) then return 1 end if
           board[nrow][ncol-nchars+1..ncol] = blank
       end for
       for m=1 to length(wmoves) do
           {?,nrow,ncol} = wmoves[m]
           warnsdorffs[nrow][ncol] += 1
       end for
   end if
   return 0

end function

procedure Hidato(sequence s, integer w, integer h, integer lim) integer y, ch, ch2, k atom t0 = time()

   s = split(s,'\n')
   width = w
   height = h
   nchars = length(sprintf(" %d",lim))
   fmt = sprintf(" %%%dd",nchars-1)
   blank = repeat('_',nchars)
   board = repeat(repeat(' ',width*nchars),height)
   knownx = repeat(0,lim)
   knowny = repeat(0,lim)
   limit = 0
   for x=1 to height do
       for y=nchars to width*nchars by nchars do
           if y>length(s[x]) then
               ch = '.'
           else
               ch = s[x][y]
           end if
           if ch='_' then
               limit += 1
           elsif ch!='.' then
               k = ch-'0'
               ch2 = s[x][y-1]
               if ch2!=' ' then
                   k += (ch2-'0')*10
                   board[x][y-1] = ch2
               end if
               knownx[k] = x
               knowny[k] = y
               limit += 1
           end if
           board[x][y] = ch
       end for
   end for
   warnsdorffs = repeat(repeat(0,width*nchars),height)
   init_warnsdorffs()
   tries = 0
   if solve(knownx[1],knowny[1],2) then
       puts(1,join(board,"\n"))
       printf(1,"\nsolution found in %d tries (%3.2fs)\n",{tries,time()-t0})
   else
       puts(1,"no solutions found\n")
   end if

end procedure

constant board1 = """

__ 33 35 __ __ .. .. ..
__ __ 24 22 __ .. .. ..
__ __ __ 21 __ __ .. ..
__ 26 __ 13 40 11 .. ..
27 __ __ __  9 __  1 ..
.. .. __ __ 18 __ __ ..
.. .. .. .. __  7 __ __
.. .. .. .. .. ..  5 __"""

Hidato(board1,8,8,40)

constant board2 = """

. 4 .
_ 7 _
1 _ _"""

Hidato(board2,3,3,7)

constant board3 = """

 1  _  _  .  .  _  _  .  .  _  _  .  .  _  _  .  .  _  _  .  .  _  _  .  .  _  _  .  .  _  _  .  .  _  _  .  .  _  _  .  .  _  _  .  .  _  _  .  . 74
 .  .  _  .  _  .  _  .  _  .  _  .  _  .  _  .  _  .  _  .  _  .  _  .  _  .  _  .  _  .  _  .  _  .  _  .  _  .  _  .  _  .  _  .  _  .  _  .  _  .
 .  .  .  _  _  .  .  _  _  .  .  _  _  .  .  _  _  .  .  _  _  .  .  _  _  .  .  _  _  .  .  _  _  .  .  _  _  .  .  _  _  .  .  _  _  .  .  _  _  ."""

Hidato(board3,50,3,74)

constant board4 = """

54 __ 60 59 __ 67 __ 69 __
__ 55 __ __ 63 65 __ 72 71
51 50 56 62 __ .. .. .. ..
__ __ __ 14 .. .. 17 __ ..
48 10 11 .. 15 __ 18 __ 22
__ 46 __ ..  3 __ 19 23 __
__ 44 __  5 __  1 33 32 __
__ 43  7 __ 36 __ 27 __ 31
42 __ __ 38 __ 35 28 __ 30"""

Hidato(board4,9,9,72)

constant board5 = """

__ 58 __ 60 __ __ 63 66 __
57 55 59 53 49 __ 65 __ 68
__  8 __ __ 50 __ 46 45 __
10  6 __ .. .. .. __ 43 70
__ 11 12 .. .. .. 72 71 __
__ 14 __ .. .. .. 30 39 __
15  3 17 __ 28 29 __ __ 40
__ __ 19 22 __ __ 37 36 __
 1 20 __ 24 __ 26 __ 34 33"""

Hidato(board5,9,9,72)

constant board6 = """

 1 __ .. .. .. __ __ .. .. .. __ __ .. .. .. __ __ .. .. .. __ __ .. .. .. __ __ .. .. .. __ __ .. .. .. __ __ .. .. .. __ __ .. .. .. 82
.. .. __ .. __ .. .. __ .. __ .. .. __ .. __ .. .. __ .. __ .. .. __ .. __ .. .. __ .. __ .. .. __ .. __ .. .. __ .. __ .. .. __ .. __ ..
.. __ .. __ .. .. __ .. __ .. .. __ .. __ .. .. __ .. __ .. .. __ .. __ .. .. __ .. __ .. .. __ .. __ .. .. __ .. __ .. .. __ .. __ .. ..
__ __ __ .. .. __ __ __ .. .. __ __ __ .. .. __ __ __ .. .. __ __ __ .. .. __ __ __ .. .. __ __ __ .. .. __ __ __ .. .. __ __ __ .. .. .."""

Hidato(board6,46,4,82)</lang>

Output:
 32 33 35 36 37  .  .  .
 31 34 24 22 38  .  .  .
 30 25 23 21 12 39  .  .
 29 26 20 13 40 11  .  .
 27 28 14 19  9 10  1  .
  .  . 15 16 18  8  2  .
  .  .  .  . 17  7  6  3
  .  .  .  .  .  .  5  4
solution found in 760 tries (0.00s)
 . 4 .
 3 7 5
 1 2 6
solution found in 10 tries (0.00s)
  1  2  3  .  .  8  9  .  . 14 15  .  . 20 21  .  . 26 27  .  . 32 33  .  . 38 39  .  . 44 45  .  . 50 51  .  . 56 57  .  . 62 63  .  . 68 69  .  . 74
  .  .  4  .  7  . 10  . 13  . 16  . 19  . 22  . 25  . 28  . 31  . 34  . 37  . 40  . 43  . 46  . 49  . 52  . 55  . 58  . 61  . 64  . 67  . 70  . 73  .
  .  .  .  5  6  .  . 11 12  .  . 17 18  .  . 23 24  .  . 29 30  .  . 35 36  .  . 41 42  .  . 47 48  .  . 53 54  .  . 59 60  .  . 65 66  .  . 71 72  .
solution found in 74 tries (0.00s)
 54 53 60 59 58 67 66 69 70
 52 55 61 57 63 65 68 72 71
 51 50 56 62 64  .  .  .  .
 49 12 13 14  .  . 17 21  .
 48 10 11  . 15 16 18 20 22
 47 46  9  .  3  2 19 23 24
 45 44  8  5  4  1 33 32 25
 41 43  7  6 36 34 27 26 31
 42 40 39 38 37 35 28 29 30
solution found in 106 tries (0.00s)
 56 58 54 60 61 62 63 66 67
 57 55 59 53 49 47 65 64 68
  9  8 52 51 50 48 46 45 69
 10  6  7  .  .  . 44 43 70
  5 11 12  .  .  . 72 71 42
  4 14 13  .  .  . 30 39 41
 15  3 17 18 28 29 38 31 40
  2 16 19 22 25 27 37 36 32
  1 20 21 24 23 26 35 34 33
solution found in 495 tries (0.00s)
  1  2  .  .  . 10 11  .  .  . 19 20  .  .  . 28 29  .  .  . 37 38  .  .  . 46 47  .  .  . 55 56  .  .  . 64 65  .  .  . 73 74  .  .  . 82
  .  .  3  .  9  .  . 12  . 18  .  . 21  . 27  .  . 30  . 36  .  . 39  . 45  .  . 48  . 54  .  . 57  . 63  .  . 66  . 72  .  . 75  . 81  .
  .  4  .  8  .  . 13  . 17  .  . 22  . 26  .  . 31  . 35  .  . 40  . 44  .  . 49  . 53  .  . 58  . 62  .  . 67  . 71  .  . 76  . 80  .  .
  5  6  7  .  . 14 15 16  .  . 23 24 25  .  . 32 33 34  .  . 41 42 43  .  . 50 51 52  .  . 59 60 61  .  . 68 69 70  .  . 77 78 79  .  .  .
solution found in 82 tries (0.02s)

PicoLisp

<lang PicoLisp>(load "@lib/simul.l")

(de hidato (Lst)

  (let Grid (grid (length (maxi length Lst)) (length Lst))
     (mapc
        '((G L)
           (mapc
              '((This Val)
                 (nond
                    (Val
                       (with (: 0 1 1) (con (: 0 1)))    # Cut off west
                       (with (: 0 1 -1) (set (: 0 1)))   # east
                       (with (: 0 -1 1) (con (: 0 -1)))  # south
                       (with (: 0 -1 -1) (set (: 0 -1))) # north
                       (set This) )
                    ((=T Val) (=: val Val)) ) )
              G L ) )
        Grid
        (apply mapcar (reverse Lst) list) )
     (let Todo
        (by '((This) (: val)) sort
           (mapcan '((Col) (filter '((This) (: val)) Col))
              Grid ) )
        (let N 1
           (with (pop 'Todo)
              (recur (N Todo)
                 (unless (> (inc 'N) (; Todo 1 val))
                    (find
                       '((Dir)
                          (with (Dir This)
                             (cond
                                ((= N (: val))
                                   (if (cdr Todo) (recurse N @) T) )
                                ((not (: val))
                                   (=: val N)
                                   (or (recurse N Todo) (=: val NIL)) ) ) ) )
                       (quote
                          west east south north
                          ((X) (or (south (west X)) (west (south X))))
                          ((X) (or (north (west X)) (west (north X))))
                          ((X) (or (south (east X)) (east (south X))))
                          ((X) (or (north (east X)) (east (north X)))) ) ) ) ) ) ) )
     (disp Grid 0
        '((This)
           (if (: val) (align 3 @) "   ") ) ) ) )</lang>

Test: <lang PicoLisp>(hidato

  (quote
     (T   33  35  T   T)
     (T   T   24  22  T)
     (T   T   T   21  T   T)
     (T   26  T   13  40  11)
     (27  T   T   T   9   T   1)
     (NIL NIL T   T   18  T   T)
     (NIL NIL NIL NIL T   7   T  T)
     (NIL NIL NIL NIL NIL NIL 5  T) ) )</lang>

Output:

   +---+---+---+---+---+---+---+---+
 8 | 32  33  35  36  37|   |   |   |
   +   +   +   +   +   +---+---+---+
 7 | 31  34  24  22  38|   |   |   |
   +   +   +   +   +   +---+---+---+
 6 | 30  25  23  21  12  39|   |   |
   +   +   +   +   +   +   +---+---+
 5 | 29  26  20  13  40  11|   |   |
   +   +   +   +   +   +   +---+---+
 4 | 27  28  14  19   9  10   1|   |
   +---+---+   +   +   +   +   +---+
 3 |   |   | 15  16  18   8   2|   |
   +---+---+---+---+   +   +   +---+
 2 |   |   |   |   | 17   7   6   3|
   +---+---+---+---+---+---+   +   +
 1 |   |   |   |   |   |   |  5   4|
   +---+---+---+---+---+---+---+---+
     a   b   c   d   e   f   g   h

Prolog

Works with SWI-Prolog and library(clpfd) written by Markus Triska.
Puzzle solved is from the Wilkipedia page : http://en.wikipedia.org/wiki/Hidato <lang Prolog>:- use_module(library(clpfd)).

hidato :- init1(Li), % skip first blank line init2(1, 1, 10, Li), my_write(Li).


init1(Li) :- Li = [ 0, 0, 0, 0, 0, 0, 0, 0, 0, 0, 0, A, 33, 35, B, C, 0, 0, 0, 0, 0, D, E, 24, 22, F, 0, 0, 0, 0, 0, G, H, I, 21, J, K, 0, 0, 0, 0, L, 26, M, 13, 40, 11, 0, 0, 0, 0, 27, N, O, P, 9, Q, 1, 0, 0, 0, 0, 0, R, S, 18, T, U, 0, 0, 0, 0, 0, 0, 0, V, 7, W, X, 0, 0, 0, 0, 0, 0, 0, 0, 5, Y, 0, 0, 0, 0, 0, 0, 0, 0, 0, 0, 0],

LV = [ A, 33, 35, B, C, D, E, 24, 22, F, G, H, I, 21, J, K, L, 26, M, 13, 40, 11, 27, N, O, P, 9, Q, 1, R, S, 18, T, U, V, 7, W, X, 5, Y],


LV ins 1..40, all_distinct(LV).

% give the constraints % Stop before the last line init2(_N, Col, Max_Col, _L) :- Col is Max_Col - 1.

% skip zeros init2(N, Lig, Col, L) :- I is N + Lig * Col, element(I, L, 0), !, V is N+1, ( V > Col -> N1 = 1, Lig1 is Lig + 1; N1 = V, Lig1 = Lig), init2(N1, Lig1, Col, L).


% skip first column init2(1, Lig, Col, L) :- !, init2(2, Lig, Col, L) .

% skip last column init2(Col, Lig, Col, L) :- !, Lig1 is Lig+1, init2(1, Lig1, Col, L).

% V5 V3 V6 % V1 V V2 % V7 V4 V8 % general case init2(N, Lig, Col, L) :- I is N + Lig * Col, element(I, L, V),

I1 is I - 1, I2 is I + 1, I3 is I - Col, I4 is I + Col, I5 is I3 - 1, I6 is I3 + 1, I7 is I4 - 1, I8 is I4 + 1,

maplist(compute_BI(L, V), [I1,I2,I3,I4,I5,I6,I7,I8], VI, BI),

sum(BI, #=, SBI),

( ((V #= 1 #\/ V #= 40) #/\ SBI #= 1) #\/ (V #\= 1 #/\ V #\= 40 #/\ SBI #= 2)) #<==> 1,

labeling([ffc, enum], [V | VI]),

N1 is N+1, init2(N1, Lig, Col, L).

compute_BI(L, V, I, VI, BI) :- element(I, L, VI), VI #= 0 #==> BI #= 0, ( VI #\= 0 #/\ (V - VI #= 1 #\/ VI - V #= 1)) #<==> BI.

% display the result my_write([0, A, B, C, D, E, F, G, H, 0 | T]) :- maplist(my_write_1, [A, B, C, D, E, F, G, H]), nl, my_write(T).

my_write([]).

my_write_1(0) :- write(' ').

my_write_1(X) :- writef('%3r', [X]).</lang>

Output:
?- hidato.
 32 33 35 36 37         
 31 34 24 22 38         
 30 25 23 21 12 39      
 29 26 20 13 40 11      
 27 28 14 19  9 10  1   
       15 16 18  8  2   
             17  7  6  3
                    5  4
true 

Python

<lang python>board = [] given = [] start = None

def setup(s):

   global board, given, start
   lines = s.splitlines()
   ncols = len(lines[0].split())
   nrows = len(lines)
   board = [[-1] * (ncols + 2) for _ in xrange(nrows + 2)]
   for r, row in enumerate(lines):
       for c, cell in enumerate(row.split()):
           if cell == "__" :
               board[r + 1][c + 1] = 0
               continue
           elif cell == ".":
               continue # -1
           else:
               val = int(cell)
               board[r + 1][c + 1] = val
               given.append(val)
               if val == 1:
                   start = (r + 1, c + 1)
   given.sort()

def solve(r, c, n, next=0):

   if n > given[-1]:
       return True
   if board[r][c] and board[r][c] != n:
       return False
   if board[r][c] == 0 and given[next] == n:
       return False
   back = 0
   if board[r][c] == n:
       next += 1
       back = n
   board[r][c] = n
   for i in xrange(-1, 2):
       for j in xrange(-1, 2):
           if solve(r + i, c + j, n + 1, next):
               return True
   board[r][c] = back
   return False

def print_board():

   d = {-1: "  ", 0: "__"}
   bmax = max(max(r) for r in board)
   form = "%" + str(len(str(bmax)) + 1) + "s"
   for r in board[1:-1]:
       print "".join(form % d.get(c, str(c)) for c in r[1:-1])

hi = """\ __ 33 35 __ __ . . . __ __ 24 22 __ . . . __ __ __ 21 __ __ . . __ 26 __ 13 40 11 . . 27 __ __ __ 9 __ 1 .

.  . __ __ 18 __ __  .
.  .  .  . __  7 __ __
.  .  .  .  .  .  5 __"""

setup(hi) print_board() solve(start[0], start[1], 1) print print_board()</lang>

Output:
 __ 33 35 __ __         
 __ __ 24 22 __         
 __ __ __ 21 __ __      
 __ 26 __ 13 40 11      
 27 __ __ __  9 __  1   
       __ __ 18 __ __   
             __  7 __ __
                    5 __

 32 33 35 36 37         
 31 34 24 22 38         
 30 25 23 21 12 39      
 29 26 20 13 40 11      
 27 28 14 19  9 10  1   
       15 16 18  8  2   
             17  7  6  3
                    5  4

Racket

Standalone

Algorithm is depth first search for each number, repeating for all numbers in ascending order. It currently runs slowish due to temporary shortcomings in untyped Racket's array indexing, but finished immediately when tested with custom 2d vector library.

<lang Racket>

  1. lang racket

(require math/array)

  1. f = not a legal position, #t = blank position

(define board

 (array
  #[#[#t 33 35 #t #t #f #f #f]
    #[#t #t 24 22 #t #f #f #f]
    #[#t #t #t 21 #t #t #f #f]
    #[#t 26 #t 13 40 11 #f #f]
    #[27 #t #t #t  9 #t  1 #f]
    #[#f #f #t #t 18 #t #t #f]
    #[#f #f #f #f #t  7 #t #t]
    #[#f #f #f #f #f #f  5 #t]]))
filters elements with the predicate, returning the element and its indices

(define (array-indices-of a f)

 (for*/list ([i (range 0 (vector-ref (array-shape a) 0))]
             [j (range 0 (vector-ref (array-shape a) 1))]
             #:when (f (array-ref a (vector i j))))
   (list (array-ref a (vector i j)) i j)))
returns a list, each element is a list of the number followed by i and j indices
sorted ascending by number

(define (goal-list v) (sort (array-indices-of v number?) (λ (a b) (< (car a) (car b)))))

every direction + start position that's on the board

(define (legal-moves a i0 j0)

 (for*/list ([i (range (sub1 i0) (+ i0 2))]
             [j (range (sub1 j0) (+ j0 2))]
             ;cartesian product -1..1 and -1..1, except 0 0
             #:when (and (not (and (= i i0) (= j j0)))
                         ;make sure it's on the board
                         (<= 0 i (sub1 (vector-ref (array-shape a) 0)))
                         (<= 0 j (sub1 (vector-ref (array-shape a) 1)))
                         ;make sure it's an actual position too (the real board isn't square)
                         (array-ref a (vector i j))))
   (cons i j)))
find path through array, returning list of coords from start to finish

(define (hidato-path a)

 ;get starting position as first goal
 (match-let ([(cons (list n i j) goals) (goal-list a)])
   (let hidato ([goals goals] [n n] [i i] [j j] [path '()])
     (match goals
       ;no more goals, return path
       ['() (reverse (cons (cons i j) path))]
       ;get next goal
       [(cons (list n-goal i-goal j-goal) _)
        (let ([move (cons i j)])
          ;already visiting a spot or taking too many moves to reach the next goal is no good
          (cond [(or (member move path) (> n n-goal)) #f]
                ;taking the right number of moves to be at the goal square is good
                ;so go to the next goal
                [(and (= n n-goal) (= i i-goal) (= j j-goal))
                 (hidato (cdr goals) n i j path)]
                ;depth first search using every legal move to find next goal
                [else (ormap (λ (m) (hidato goals (add1 n) (car m) (cdr m) (cons move path)))
                             (legal-moves a i j))]))]))))
take a path and insert it into the array

(define (put-path a path)

 (let ([a (array->mutable-array a)])
   (for ([n (range 1 (add1 (length path)))] [move path])
     (array-set! a (vector (car move) (cdr move)) n))
   a))
main function

(define (hidato board) (put-path board (hidato-path board))) </lang>

Output:
> (hidato board)
(mutable-array
 #[#[32 33 35 36 37 #f #f #f]
   #[31 34 24 22 38 #f #f #f]
   #[30 25 23 21 12 39 #f #f]
   #[29 26 20 13 40 11 #f #f]
   #[27 28 14 19 9 10 1 #f]
   #[#f #f 15 16 18 8 2 #f]
   #[#f #f #f #f 17 7 6 3]
   #[#f #f #f #f #f #f 5 4]])

Using Hidato Family Solver from Numbrix

This solution uses the module "hidato-family-solver.rkt" from Solve a Numbrix puzzle#Racket. The difference between the two is essentially the neighbourhood function.

<lang racket>#lang racket (require "hidato-family-solver.rkt")

(define moore-neighbour-offsets

 '((+1 0) (-1 0) (0 +1) (0 -1) (+1 +1) (-1 -1) (-1 +1) (+1 -1)))

(define solve-hidato (solve-hidato-family moore-neighbour-offsets))

(displayln

(puzzle->string
 (solve-hidato
  #(#( 0 33 35  0  0)
    #( 0  0 24 22  0)
    #( 0  0  0 21  0  0)
    #( 0 26  0 13 40 11)
    #(27  0  0  0  9  0  1)
    #( _  _  0  0 18  0  0)
    #( _  _  _  _  0  7  0  0)
    #( _  _  _  _  _  _  5  0)))))

</lang>

Output:
32 33 35 36 37  _  _  _
31 34 24 22 38  _  _  _
30 25 23 21 12 39  _  _
29 26 20 13 40 11  _  _
27 28 14 19  9 10  1  _
 _  _ 15 16 18  8  2  _
 _  _  _  _ 17  7  6  3
 _  _  _  _  _  _  5  4

REXX

Programming note:   the coördinates for the cells used are the same as an   X Y   grid, that is,
the bottom left-most cell is   1 1   and the tenth cell on row 2 is   2 10
If any marker is negative, then it's assumed to be a Numbrix puzzle (and the absolute value is used).
Over half of the REXX program deals with validating the input and displaying the puzzle.

Hidato   and   Numbrix   are registered trademarks. <lang rexx>/*REXX program solves a Numbrix (R) puzzle, it also displays the puzzle and solution. */ maxR=0; maxC=0; maxX=0; minR=9e9; minC=9e9; minX=9e9; cells=0; @.= parse arg xxx; PZ='Hidato puzzle' /*get the cell definitions from the CL.*/ xxx=translate(xxx, , "/\;:_", ',') /*also allow other characters as comma.*/

              do  while xxx\=;  parse var  xxx    r c   marks  ','  xxx
                  do  while marks\=;          _=@.r.c
                  parse var marks  x  marks
                  if datatype(x,'N')  then do;  x=x/1                     /*normalize X*/
                                                if x<0  then PZ= 'Numbrix puzzle'
                                                x=abs(x)                  /*use  │x│   */
                                           end
                  minR=min(minR,r);  maxR=max(maxR,r); minC=min(minC,c); maxC=max(maxC,c)
                  if x==1   then do;  !r=r;  !c=c;  end              /*the START cell. */
                  if _\== then call err "cell at" r c 'is already occupied with:'  _
                  @.r.c=x;   c=c+1;    cells=cells+1                 /*assign a mark.  */
                  if x==.              then iterate                  /*is a hole?  Skip*/
                  if \datatype(x,'W')  then call err 'illegal marker specified:' x
                  minX=min(minX,x);    maxX=max(maxX,x)              /*min and max  X. */
                  end   /*while marks¬= */
              end       /*while xxx  ¬= */

call show /* [↓] is used for making fast moves. */ Nr = '0 1 0 -1 -1 1 1 -1' /*possible row for the next move. */ Nc = '1 0 -1 0 1 -1 1 -1' /* " column " " " " */ pMoves=words(Nr) -4*(left(PZ,1)=='N') /*is this to be a Numbrix puzzle ? */

 do i=1  for pMoves;   Nr.i=word(Nr,i);   Nc.i=word(Nc,i);   end     /*for fast moves. */

if \next(2,!r,!c) then call err 'No solution possible for this' PZ "puzzle." say 'A solution for the' PZ "exists."; say; call show exit /*stick a fork in it, we're all done. */ /*──────────────────────────────────────────────────────────────────────────────────────*/ err: say; say '***error*** (from' PZ"): " arg(1); say; exit 13 /*──────────────────────────────────────────────────────────────────────────────────────*/ next: procedure expose @. Nr. Nc. cells pMoves; parse arg #,r,c; ##=#+1

          do t=1  for pMoves                                   /* [↓]  try some moves. */
          parse value  r+Nr.t c+Nc.t  with nr nc               /*next move coördinates.*/
          if @.nr.nc==.  then do;                @.nr.nc=#     /*let's try this move.  */
                              if #==cells        then leave    /*is this the last move?*/
                              if next(##,nr,nc)  then return 1
                              @.nr.nc=.                        /*undo the above move.  */
                              iterate                          /*go & try another move.*/
                              end
          if @.nr.nc==#  then do                               /*this a fill-in move ? */
                              if #==cells        then return 1 /*this is the last move.*/
                              if next(##,nr,nc)  then return 1 /*a fill-in move.       */
                              end
          end   /*t*/
     return 0                                                  /*this ain't working.   */

/*──────────────────────────────────────────────────────────────────────────────────────*/ show: if maxR<1 | maxC<1 then call err 'no legal cell was specified.'

     if minX<1           then call err  'no  1  was specified for the puzzle start'
     w=max(2,length(cells));  do    r=maxR  to minR  by -1; _=
                                 do c=minC  to maxC;        _=_ right(@.r.c,w); end /*c*/
                              say _
                              end   /*r*/
     say;    return</lang>

output   when using the following as input:
1 7 5 .\2 5 . 7 . .\3 3 . . 18 . .\4 1 27 . . . 9 . 1\5 1 . 26 . 13 40 11\6 1 . . . 21 . .\7 1 . . 24 22 .\8 1 . 33 35 . .

  . 33 35  .  .
  .  . 24 22  .
  .  .  . 21  .  .
  . 26  . 13 40 11
 27  .  .  .  9  .  1
        .  . 18  .  .
              .  7  .  .
                    5  .


A solution for the Hidato puzzle exists.

 32 33 35 36 37
 31 34 24 22 38
 30 25 23 21 12 39
 29 26 20 13 40 11
 27 28 14 19  9 10  1
       15 16 18  8  2
             17  7  6  3
                    5  4

Ruby

Without Warnsdorff

The following class provides functionality for solving a hidato problem: <lang ruby># Solve a Hidato Puzzle

class Hidato

 Cell = Struct.new(:value, :used, :adj)
 ADJUST = [[-1, -1], [-1, 0], [-1, 1], [0, -1], [0, 1], [1, -1], [1, 0], [1, 1]]
 
 def initialize(board, pout=true)
   @board = []
   board.each_line do |line|
     @board << line.split.map{|n| Cell[Integer(n), false] rescue nil} + [nil]
   end
   @board << []                                # frame (Sentinel value : nil)
   @board.each_with_index do |row, x|
     row.each_with_index do |cell, y|
       if cell
         @sx, @sy = x, y  if cell.value==1     # start position
         cell.adj = ADJUST.map{|dx,dy| [x+dx,y+dy]}.select{|xx,yy| @board[xx][yy]}
       end
     end
   end
   @xmax = @board.size - 1
   @ymax = @board.map(&:size).max - 1
   @end  = @board.flatten.compact.size
   puts to_s('Problem:')  if pout
 end
 
 def solve
   @zbl = Array.new(@end+1, false)
   @board.flatten.compact.each{|cell| @zbl[cell.value] = true}
   puts (try(@board[@sx][@sy], 1) ? to_s('Solution:') : "No solution")
 end
 
 def try(cell, seq_num)
   return true  if seq_num > @end
   return false if cell.used
   value = cell.value
   return false if value > 0 and value != seq_num
   return false if value == 0 and @zbl[seq_num]
   cell.used = true
   cell.adj.each do |x, y|
     if try(@board[x][y], seq_num+1)
       cell.value = seq_num
       return true
     end
   end
   cell.used = false
 end
 
 def to_s(msg=nil)
   str = (0...@xmax).map do |x|
     (0...@ymax).map{|y| "%3s" % ((c=@board[x][y]) ? c.value : c)}.join
   end
   (msg ? [msg] : []) + str + [""]
 end

end</lang>

Test: <lang ruby># Which may be used as follows to solve Evil Case 1: board1 = <<EOS

 .  4
 0  7  0
 1  0  0

EOS Hidato.new(board1).solve

  1. Which may be used as follows to solve this tasks example:

board2 = <<EOS

 0 33 35  0  0
 0  0 24 22  0
 0  0  0 21  0  0
 0 26  0 13 40 11
27  0  0  0  9  0  1
 .  .  0  0 18  0  0
 .  .  .  .  0  7  0  0
 .  .  .  .  .  .  5  0

EOS Hidato.new(board2).solve

  1. Which may be used as follows to solve The Snake in the Grass:

board3 = <<EOS

 1  0  0  .  .  0  0  .  .  0  0  .  .  0  0  .  .  0  0  .  .  0  0  .  .  0  0  .  .  0  0  .  .  0  0  .  .  0  0  .  .  0  0  .  .  0  0  .  . 74
 .  .  0  .  0  .  0  .  0  .  0  .  0  .  0  .  0  .  0  .  0  .  0  .  0  .  0  .  0  .  0  .  0  .  0  .  0  .  0  .  0  .  0  .  0  .  0  .  0  .
 .  .  .  0  0  .  .  0  0  .  .  0  0  .  .  0  0  .  .  0  0  .  .  0  0  .  .  0  0  .  .  0  0  .  .  0  0  .  .  0  0  .  .  0  0  .  .  0  0  .

EOS t0 = Time.now Hidato.new(board3).solve puts " #{Time.now - t0} sec"</lang>

Output:
Problem:
     4   
  0  7  0
  1  0  0

Solution:
     4   
  3  7  5
  1  2  6

Problem:
  0 33 35  0  0         
  0  0 24 22  0         
  0  0  0 21  0  0      
  0 26  0 13 40 11      
 27  0  0  0  9  0  1   
        0  0 18  0  0   
              0  7  0  0
                    5  0

Solution:
 32 33 35 36 37         
 31 34 24 22 38         
 30 25 23 21 12 39      
 29 26 20 13 40 11      
 27 28 14 19  9 10  1   
       15 16 18  8  2   
             17  7  6  3
                    5  4

Problem:
  1  0  0        0  0        0  0        0  0        0  0        0  0        0  0        0  0        0  0        0  0        0  0        0  0       74
        0     0     0     0     0     0     0     0     0     0     0     0     0     0     0     0     0     0     0     0     0     0     0     0   
           0  0        0  0        0  0        0  0        0  0        0  0        0  0        0  0        0  0        0  0        0  0        0  0   

Solution:
  1  2  3        8  9       14 15       20 21       26 27       32 33       38 39       44 45       50 51       56 57       62 63       68 69       74
        4     7    10    13    16    19    22    25    28    31    34    37    40    43    46    49    52    55    58    61    64    67    70    73   
           5  6       11 12       17 18       23 24       29 30       35 36       41 42       47 48       53 54       59 60       65 66       71 72   

 40.198299 sec

With Warnsdorff

I modify method as follows to implement Warnsdorff like <lang ruby># Solve a Hidato Like Puzzle with Warnsdorff like logic applied

class HLPsolver

 attr_reader :board
 Cell = Struct.new(:value, :used, :adj)
 
 def initialize(board, pout=true)
   @board = []
   frame = ADJACENT.flatten.map(&:abs).max
   board.each_line do |line|
     @board << line.split.map{|n| Cell[Integer(n), false] rescue nil} + [nil]*frame
   end
   frame.times {@board << []}                  # frame (Sentinel value : nil)
   @board.each_with_index do |row, x|
     row.each_with_index do |cell, y|
       if cell
         @sx, @sy = x, y  if cell.value==1     # start position
         cell.adj = ADJACENT.map{|dx,dy| [x+dx,y+dy]}.select{|xx,yy| @board[xx][yy]}
       end
     end
   end
   @xmax = @board.size - frame
   @ymax = @board.map(&:size).max - frame
   @end  = @board.flatten.compact.size
   @format = " %#{@end.to_s.size}s"
   puts to_s('Problem:')  if pout
 end
 
 def solve
   @zbl = Array.new(@end+1, false)
   @board.flatten.compact.each{|cell| @zbl[cell.value] = true}
   puts (try(@board[@sx][@sy], 1) ? to_s('Solution:') : "No solution")
 end
 
 def try(cell, seq_num)
   value = cell.value
   return false if value > 0 and value != seq_num
   return false if value == 0 and @zbl[seq_num]
   cell.used = true
   if seq_num == @end
     cell.value = seq_num
     return true
   end
   a = []
   cell.adj.each_with_index do |(x, y), n|
     cl = @board[x][y]
     a << [wdof(cl.adj)*10+n, x, y]  unless cl.used
   end
   a.sort.each do |key, x, y|
     if try(@board[x][y], seq_num+1)
       cell.value = seq_num
       return true
     end
   end
   cell.used = false
 end
 
 def wdof(adj)
   adj.count {|x,y| not @board[x][y].used}
 end
 
 def to_s(msg=nil)
   str = (0...@xmax).map do |x|
     (0...@ymax).map{|y| @format % ((c=@board[x][y]) ? c.value : c)}.join
   end
   (msg ? [msg] : []) + str + [""]
 end

end</lang> Which may be used as follows to solve Hidato Puzzles: <lang ruby>require 'HLPsolver'

ADJACENT = [[-1, -1], [-1, 0], [-1, 1], [0, -1], [0, 1], [1, -1], [1, 0], [1, 1]]

  1. solve Evil Case 1:

board1 = <<EOS

 .  4
 0  7  0
 1  0  0

EOS HLPsolver.new(board1).solve

boardx = <<EOS

 0  0  0  0  0  0  0  0
 0  0  0  0  0  0  0  0
 0  0  0  0  0  0  0  0
 0  1  0  0  0  0  0  0
 0  0  0  0  0  0  0  0
 0  0  0  0  0  0  0  0
 0  0  0  0  0  0  0  0
 0  0  0  0  0  0  0  0

EOS HLPsolver.new(boardx).solve

  1. solve this tasks example:

board2 = <<EOS

 0 33 35  0  0
 0  0 24 22  0
 0  0  0 21  0  0
 0 26  0 13 40 11
27  0  0  0  9  0  1
 .  .  0  0 18  0  0
 .  .  .  .  0  7  0  0
 .  .  .  .  .  .  5  0

EOS HLPsolver.new(board2).solve

  1. solve The Snake in the Grass:

board3 = <<EOS

 1  0  0  .  .  0  0  .  .  0  0  .  .  0  0  .  .  0  0  .  .  0  0  .  .  0  0  .  .  0  0  .  .  0  0  .  .  0  0  .  .  0  0  .  .  0  0  .  . 74
 .  .  0  .  0  .  0  .  0  .  0  .  0  .  0  .  0  .  0  .  0  .  0  .  0  .  0  .  0  .  0  .  0  .  0  .  0  .  0  .  0  .  0  .  0  .  0  .  0  .
 .  .  .  0  0  .  .  0  0  .  .  0  0  .  .  0  0  .  .  0  0  .  .  0  0  .  .  0  0  .  .  0  0  .  .  0  0  .  .  0  0  .  .  0  0  .  .  0  0  .

EOS t0 = Time.now HLPsolver.new(board3).solve puts " #{Time.now - t0} sec"</lang>

Which produces:

Problem:
   4  
 0 7 0
 1 0 0

Solution:
   4  
 3 7 5
 1 2 6

Problem:
  0  0  0  0  0  0  0  0
  0  0  0  0  0  0  0  0
  0  0  0  0  0  0  0  0
  0  1  0  0  0  0  0  0
  0  0  0  0  0  0  0  0
  0  0  0  0  0  0  0  0
  0  0  0  0  0  0  0  0
  0  0  0  0  0  0  0  0

Solution:
 33 34 36 37 41 42 43 44
 32 35 38 40 56 55 46 45
  2 31 39 57 59 60 54 47
  3  1 30 58 61 62 53 48
  4  6 18 29 63 64 52 49
  5  7 17 19 28 51 50 25
  8 11 13 16 20 27 26 24
  9 10 12 14 15 21 22 23

Problem:
  0 33 35  0  0         
  0  0 24 22  0         
  0  0  0 21  0  0      
  0 26  0 13 40 11      
 27  0  0  0  9  0  1   
        0  0 18  0  0   
              0  7  0  0
                    5  0

Solution:
 32 33 35 36 37         
 31 34 24 22 38         
 30 25 23 21 12 39      
 29 26 20 13 40 11      
 27 28 14 19  9 10  1   
       15 16 18  8  2   
             17  7  6  3
                    5  4

Problem:
  1  0  0        0  0        0  0        0  0        0  0        0  0        0  0        0  0        0  0        0  0        0  0        0  0       74
        0     0     0     0     0     0     0     0     0     0     0     0     0     0     0     0     0     0     0     0     0     0     0     0   
           0  0        0  0        0  0        0  0        0  0        0  0        0  0        0  0        0  0        0  0        0  0        0  0   

Solution:
  1  2  3        8  9       14 15       20 21       26 27       32 33       38 39       44 45       50 51       56 57       62 63       68 69       74
        4     7    10    13    16    19    22    25    28    31    34    37    40    43    46    49    52    55    58    61    64    67    70    73   
           5  6       11 12       17 18       23 24       29 30       35 36       41 42       47 48       53 54       59 60       65 66       71 72   

 0.003001 sec

HLPsolver may be used to solve Knight's tour:

Seed7

<lang seed7>$ include "seed7_05.s7i";

var set of integer: given is {}; var array array integer: board is 0 times 0 times 0; var integer: startRow is 0; var integer: startColumn is 0;

const proc: setup (in array string: input) is func

 local
   var integer: r is 0;
   var integer: c is 0;
   var array string: row is 0 times "";
   var string: cell is "";
   var integer: value is 0;
 begin
   board := (length(input) + 2) times 0 times 0;
   for key r range input do
     row := split(input[r], " ");
     board[r + 1] := (length(row) + 2) times - 1;
     for key c range row do
       cell := row[c];
       if cell = "_" then
         board[r + 1][c + 1] := 0;
       elsif cell[1] in {'0' .. '9'} then
         value := integer parse cell;
         board[r + 1][c + 1] := value;
         incl(given, value);
         if value = 1 then
           startRow := r + 1;
           startColumn := c + 1;
         end if;
       end if;
     end for;
   end for;
   board[1] := (length(row) + 2) times - 1;
   board[length(input) + 2] := (length(row) + 2) times - 1;
 end func;

const func boolean: solve (in integer: r, in integer: c, in integer: n) is func

 result
   var boolean: solved is FALSE;
 local
   var integer: back is 0;
   var integer: i is 0;
   var integer: j is 0;
 begin
   if n > max(given) then
     solved := TRUE;
   elsif board[r][c] = 0 and n not in given or board[r][c] = n then
     back := board[r][c];
     board[r][c] := n;
     for i range -1 to 1 until solved do
       for j range -1 to 1 until solved do
         solved := solve(r + i, c + j, n + 1);
       end for;
     end for;
     if not solved then
       board[r][c] := back;
     end if;
   end if;
 end func;

const proc: printBoard is func

 local
   var integer: r is 0;
   var integer: c is 0;
 begin
   for key r range board do
     for c range board[r] do
       if c = -1 then
         write(" . ");
       elsif c > 0 then
         write(c lpad 2 <& " ");
       else
         write("__ ");
       end if;
     end for;
     writeln;
   end for;
 end func;

const proc: main is func

 local
   const array string: input is [] ("_ 33 35 _ _ . . .",
                                    "_ _ 24 22 _ . . .",
                                    "_ _ _ 21 _ _ . .",
                                    "_ 26 _ 13 40 11 . .",
                                    "27 _ _ _ 9 _ 1 .",
                                    ". . _ _ 18 _ _ .",
                                    ". . . . _ 7 _ _",
                                    ". . . . . . 5 _");
 begin
   setup(input);
   printBoard;
   writeln;
   if solve(startRow, startColumn, 1) then
     writeln("Found:");
     printBoard;
   end if;
 end func;</lang>
Output:
 .  .  .  .  .  .  .  .  .  . 
 . __ 33 35 __ __  .  .  .  . 
 . __ __ 24 22 __  .  .  .  . 
 . __ __ __ 21 __ __  .  .  . 
 . __ 26 __ 13 40 11  .  .  . 
 . 27 __ __ __  9 __  1  .  . 
 .  .  . __ __ 18 __ __  .  . 
 .  .  .  .  . __  7 __ __  . 
 .  .  .  .  .  .  .  5 __  . 
 .  .  .  .  .  .  .  .  .  . 

Found:
 .  .  .  .  .  .  .  .  .  . 
 . 32 33 35 36 37  .  .  .  . 
 . 31 34 24 22 38  .  .  .  . 
 . 30 25 23 21 12 39  .  .  . 
 . 29 26 20 13 40 11  .  .  . 
 . 27 28 14 19  9 10  1  .  . 
 .  .  . 15 16 18  8  2  .  . 
 .  .  .  .  . 17  7  6  3  . 
 .  .  .  .  .  .  .  5  4  . 
 .  .  .  .  .  .  .  .  .  . 

Tcl

<lang tcl>proc init {initialConfiguration} {

   global grid max filled
   set max 1
   set y 0
   foreach row [split [string trim $initialConfiguration "\n"] "\n"] {

set x 0 set rowcontents {} foreach cell $row { if {![string is integer -strict $cell]} {set cell -1} lappend rowcontents $cell set max [expr {max($max, $cell)}] if {$cell > 0} { dict set filled $cell [list $y $x] } incr x } lappend grid $rowcontents incr y

   }

}

proc findseps {} {

   global max filled
   set result {}
   for {set i 1} {$i < $max-1} {incr i} {

if {[dict exists $filled $i]} { for {set j [expr {$i+1}]} {$j <= $max} {incr j} { if {[dict exists $filled $j]} { if {$j-$i > 1} { lappend result [list $i $j [expr {$j-$i}]] } break } } }

   }
   return [lsort -integer -index 2 $result]

}

proc makepaths {sep} {

   global grid filled
   lassign $sep from to len
   lassign [dict get $filled $from] y x
   set result {}
   foreach {dx dy} {-1 -1  -1 0  -1 1  0 -1  0 1  1 -1  1 0  1 1} {

discover [expr {$x+$dx}] [expr {$y+$dy}] [expr {$from+1}] $to \ [list [list $from $x $y]] $grid

   }
   return $result

} proc discover {x y n limit path model} {

   global filled
   # Check for illegal
   if {[lindex $model $y $x] != 0} return
   upvar 1 result result
   lassign [dict get $filled $limit] ly lx
   # Special case
   if {$n == $limit-1} {

if {abs($x-$lx)<=1 && abs($y-$ly)<=1 && !($lx==$x && $ly==$y)} { lappend result [lappend path [list $n $x $y] [list $limit $lx $ly]] } return

   }
   # Check for impossible
   if {abs($x-$lx) > $limit-$n || abs($y-$ly) > $limit-$n} return
   # Recursive search
   lappend path [list $n $x $y]
   lset model $y $x $n
   incr n
   foreach {dx dy} {-1 -1  -1 0  -1 1  0 -1  0 1  1 -1  1 0  1 1} {

discover [expr {$x+$dx}] [expr {$y+$dy}] $n $limit $path $model

   }

}

proc applypath {path} {

   global grid filled
   puts "Found unique path for [lindex $path 0 0] -> [lindex $path end 0]"
   foreach cell [lrange $path 1 end-1] {

lassign $cell n x y lset grid $y $x $n dict set filled $n [list $y $x]

   }

}

proc printgrid {} {

   global grid max
   foreach row $grid {

foreach cell $row { puts -nonewline [format " %*s" [string length $max] [expr { $cell==-1 ? "." : $cell }]] } puts ""

   }

}

proc solveHidato {initialConfiguration} {

   init $initialConfiguration
   set limit [llength [findseps]]
   while {[llength [set seps [findseps]]] && [incr limit -1]>=0} {

foreach sep $seps { if {[llength [set paths [makepaths $sep]]] == 1} { applypath [lindex $paths 0] break } }

   }
   puts ""
   printgrid

}</lang> Demonstrating (dots are “outside” the grid, and zeroes are the cells to be filled in): <lang tcl>solveHidato "

    0  33  35   0   0   .   .   .
    0   0  24  22   0   .   .   .
    0   0   0  21   0   0   .   .
    0  26   0  13  40  11   .   .
   27   0   0   0   9   0   1   .
    .   .   0   0  18   0   0   .
    .   .   .   .   0   7   0   0 
    .   .   .   .   .   .   5   0 

"</lang>

Output:
Found unique path for 5 -> 7
Found unique path for 7 -> 9
Found unique path for 9 -> 11
Found unique path for 11 -> 13
Found unique path for 33 -> 35
Found unique path for 18 -> 21
Found unique path for 1 -> 5
Found unique path for 35 -> 40
Found unique path for 22 -> 24
Found unique path for 24 -> 26
Found unique path for 27 -> 33
Found unique path for 13 -> 18

 32 33 35 36 37  .  .  .
 31 34 24 22 38  .  .  .
 30 25 23 21 12 39  .  .
 29 26 20 13 40 11  .  .
 27 28 14 19  9 10  1  .
  .  . 15 16 18  8  2  .
  .  .  .  . 17  7  6  3
  .  .  .  .  .  .  5  4

More complex cases are solvable with an extended version of this code, though that has more onerous version requirements.

zkl

Translation of: Python

<lang zkl>hi:= // 0==empty cell, X==not a cell

  1. <<<
"0  33  35   0   0   X   X   X
 0   0  24  22   0   X   X   X
 0   0   0  21   0   0   X   X
 0  26   0  13  40  11   X   X
27   0   0   0   9   0   1   X
 X   X   0   0  18   0   0   X
 X   X   X   X   0   7   0   0
 X   X   X   X   X   X   5   0";
  1. <<<

board,given,start:=setup(hi); print_board(board); solve(board,given, start.xplode(), 1); println(); print_board(board);</lang> <lang zkl>fcn print_board(board){

  d:=D(-1,"  ", 0,"__");
  foreach r in (board[1,-1]){
     r[1,-1].pump(String,'wrap(c){ "%2s ".fmt(d.find(c,c)) }).println();
  }

} fcn setup(s){

  lines:=s.split("\n");
  ncols,nrows:=lines[0].split().len(),lines.len();
  board:=(nrows+2).pump(List(), (ncols+2).pump(List(),-1).copy);
  given,start:=List(),Void;
  foreach r,row in (lines.enumerate()){
     foreach c,cell in (row.split().enumerate()){
        if(cell=="X") continue;   // X == not in play, leave at -1

val:=cell.toInt(); board[r+1][c+1]=val; given.append(val); if(val==1) start=T(r+1,c+1);

     }
  }
  return(board,given.filter().sort(),start);

} fcn solve(board,given, r,c,n, next=0){

  if(n>given[-1])                       return(True);
  if(board[r][c] and board[r][c]!=n)    return(False);
  if(board[r][c]==0 and given[next]==n) return(False);

  back:=0;
  if(board[r][c]==n){ next+=1; back=n; }

  board[r][c]=n;
  foreach i,j in ([-1..1],[-1..1]){
     if(solve(board,given, r+i,c+j,n+1, next)) return(True);
  }
  board[r][c]=back;
  False

}</lang>

Output:
__ 33 35 __ __          
__ __ 24 22 __          
__ __ __ 21 __ __       
__ 26 __ 13 40 11       
27 __ __ __  9 __  1    
      __ __ 18 __ __    
            __  7 __ __ 
                   5 __ 

32 33 35 36 37          
31 34 24 22 38          
30 25 23 21 12 39       
29 26 20 13 40 11       
27 28 14 19  9 10  1    
      15 16 18  8  2    
            17  7  6  3 
                   5  4